Vous êtes sur la page 1sur 107

curso de post-grado para profesores

tica
especialidad en matema
GEOMETRIA EUCLIDEANA
Equipo de Dise
no:
Nahomy Jhopselyn Hernandez Cruz
Gabriel Alexander Chicas Reyes
Eduardo Arnoldo Aguilar Ca
nas
Hector Enmanuel Alberti Arroyo
Ernesto Americo Hidalgo Castellanos
Juan Agustn Cuadra
Claudia Patricia Corcio Lopez de Beltran
Carlos Mauricio Canjura Linares
Oscar Armando Hernandez Morales
Aar
on Ernesto Ramrez Flores
21 de febrero de 2010

Indice

1. Angulos
entre paralelas.

2. Tri
angulos: Teoremas Fundamentales.

3. Congruencia de Tri
angulos.

16

4. Cuadril
ateros: Clasificaci
on y Propiedades.

23

5. Angulos
en la Circunferencia.

29

6. Teorema de Thales y su recproco. Semejanza de Tri


angulos.

45

7. Puntos y Rectas Notables del Tri


angulo.

57

8. Soluci
on a Problemas Selectos.

71

1.

Angulos
entre paralelas.

ANGULOS
Definimos como angulo a la figura geometrica formada por dos rayos (o semirrectas) distintas
que tienen el mismo origen. Ese origen se llama vertice del angulo. Al angulo de vertice O y
rayos OA y OB se le denota AOB.
Dos angulos AOB y BOC son adyacentes si y solo si tienen un lado com
un OB y los lados
no comunes OA y OC estan en semiplanos distintos, determinados por el lado com
un.
Bisectriz de un angulo es la semirrecta que lo divide en dos angulos adyacentes iguales.
Dos angulos son:
Congruentes o Iguales: si tienen igual medida.
Suplementarios: si su suma es 180.
Complementarios: si su suma es 90.
Por otra parte, dos rectas en el plano pueden ser secantes o paralelas,1 dependiendo si se cortan
o no; ademas, si las rectas son secantes, el punto de corte es u
nico, y definen cuatro angulos,
que se agrupan por parejas en angulos opuestos por el vertice (las parejas de angulos tales que
uno esta formado por la prolongacion de los lados del otro).
Los angulos opuestos por el vertice son iguales (Justifique), por lo que dos rectas secantes forman cuatro angulos que definen dos parejas de angulos iguales, y si tomamos un miembro de
cada pareja, se tienen dos angulos suplementarios. En particular, si las rectas son secantes y
forman cuatro angulos iguales, seran llamadas rectas perpendiculares,2 y los angulos as generados son llamados angulos rectos. Y como es muy conocido, un angulo agudo es aquel cuya
medida es menor a la de un angulo recto, y un angulo obtuso es aquel cuya medida es mayor
que un angulo recto; en particular, un angulo obtuso sera llamado angulo llano si su medida es
el doble que la de un angulo recto.

ANGULOS
ENTRE PARALELAS
Al intersecar un par de rectas paralelas por una recta llamada transversal o secante, se forman
los siguientes tipos de angulo:

Angulos
Correspondientes: Son dos angulos no adyacentes situados en el mismo lado de
la secante, uno en el interior y otro en el exterior de las paralelas.

Angulos
Alternos Internos: Son dos angulos no adyacentes situados en el interior de las
paralelas, y en distintos lado de la secante.
1
2

Si la recta AB es paralela a la recta CD, se denota AB k CD.


Si la recta AB es perpendicular a la recta CD, se denota AB CD.


Angulos
Alternos Externos: Son dos angulos no adyacentes situados en el exterior de las
paralelas, y en distintos lado de la secante.

Angulos
Conjugados: Son los angulos no adyacentes situados uno en el interior y el otro
en el exterior de las rectas paralelas y del mismo lado de la secante.
Las propiedades fundamentales de los angulos entre paralelas son:
1. Los angulos correspondientes son iguales entre s.
2. Los angulos alternos internos son iguales entre s.
3. Los angulos alternos externos son iguales entre s.
4. Los angulos conjugados son suplementarios.

Figura 1: Angulos
entre las rectas paralelas L1 y L2 .

Ejercicios
1. Tres angulos adyacentes forman un semiplano y tienen sus medidas proporcionales a los
n
umeros 5, 7 y 8. Hallar la medida del menor angulo.
2. Demostrar que las bisectrices de dos angulos suplementarios son perpendiculares.

3. En la figura adjunta, L1 k L2 y
L3 k L4 . Calcular x.

4. Con ayuda de la figura 2, demuestre que: Si L1 k L2 entonces = + .

Figura 2
5. En la figura 3, AB k F G. Hallar el angulo x si el AM F = 90y el M AB = 110.

Figura 3
6. Calcular el OP Q, si OP es bisectriz del angulo O, L1 k L2 y P Q L1 . Ver figura 4.

Figura 4

7. En la figura 5, L1 k L2 y L3 k L4 , calcular .

Figura 5
8. En la figura 6, calcular x, si L1 k L2 .

Figura 6

9. Calcular la medida del grafico anexo, si las


rectas L1 y L2 son paralelas.

10. En la figura 7, L1 k L2 y L3 k L4 . Hallar el valor del angulo .

Figura 7
11. Sea AOB = 24, en la region exterior a dicho angulo se traza el rayo OC. Hallar la
medida del angulos formado por las bisectrices de los angulos AOC y BOC.
12. Del grafico 8, calcular y, cuando x tome su maximo valor entero.

Figura 8

2.

Tri
angulos: Teoremas Fundamentales.

TEOREMAS FUNDAMENTALES EN TODO TRIANGULO.


Diremos que tres puntos que pertenecen a una misma recta son puntos colineales; de manera
analoga, si tres rectas pasan por un mismo punto, seran llamadas rectas concurrentes. Si tomamos al azar tres puntos en el plano, en muy raras ocasiones estos puntos estaran alineados,3
y diremos entonces que son los vertices de un triangulo; analogamente sucede con las rectas,
tres rectas por lo general no concurren, y la figura geometrica que estas definen es tambien un
triangulo.4 Una definicion completa para nuestros intereses es la siguiente:
Definici
on de Tri
angulo. Si A, B y C son tres puntos cualesquiera no colineales (Ver figura
9), entonces la reunion se los segmentos AB, BC y AC se llama triangulo ABC y se denota
por 4ABC. Los puntos A, B y C se llaman vertices y los segmentos AB, BC y AC se llaman
lados. Simbolicamente: 4ABC = AB BC AC. Todo triangulo ABC determina tres angulos
internos o interiores: ABC, ACB y BAC, y se llamara angulo externo o exterior, al
angulo determinado por un lado y la prolongacion del lado adyacente, en la figura anterior ,
y son angulos exteriores.

Figura 9: Elementos del Triangulo


Dado el 4ABC, se tiene que AB + BC + CA = p = 2s, donde p es llamado el permetro y s
el semipermetro del triangulo. Para abreviar, suele asociarse a cada vertice un lado opuesto,
y viceversa, por ejemplo, el lado opuesto de A es BC, y es frecuente que se denote por a;
analogamente b = CA, c = AB.
Teorema 1: En todo triangulo, la medida de un angulo exterior es igual a la suma de las
medidas de dos angulos interiores del triangulo no adyacentes a el.
La demostracion de este teorema se basa en las relaciones de angulos entre paralelas; se deja
al lector que haga la demostracion (Sugerencia: por un vertice, trace una recta paralela al lado
3

En teora de probabilidades, la probabilidad que esto ocurra es cero!


El termino m
as riguroso para esta figura es tril
atero. En este caso, habra que hacer una consideraci
on: si
hay un par de rectas paralelas, el tril
atero definido ya no es normal seg
un nuestro sentido com
un, sin embargo,
sigue siendo un tril
atero!
4

opuesto)
Corolario: En todo triangulo, la suma de las medidas de sus tres angulos internos es igual a
180.
Teorema 2: Desigualdad Triangular. En todo triangulo, la longitud de uno de sus lados
esta comprendido entre la suma y la diferencia de los otros dos.
Sin ser muy rigurosos, suponga que dado el segmento AB se traza con centro en A una circunferencia de radio r1 , y con centro en B una circunferencia de radio r2 ; si AB < r1 + r2 ,
las circunferencias se cortaran en dos puntos, y cualquiera de ellos puede ser el vertice C,
as AB < BC + CA; en cambio, si AB = r1 + r2 o peor a
un, si AB > r1 + r2 , la construccion
del 4ABC no es posible.
La Desigualdad Triangular es un resultado fundamental, a partir de esta y de su modelo de
demostracion se generan los Criterios de Congruencia de Triangulos; a groso modo, si dadas
ciertas condiciones, la construccion de una figura geometrica (un triangulo en particular) queda
determinada de manera u
nica, entonces dos figuras que reunen las mismas condiciones seran
llamadas figuras congruentes.
As, si se tienen tres segmentos (cuyas longitudes cumplen la desigualdad triangular), dejando
uno fijo y construyendo las circunferencias con centros en los extremos de este segmento y
radios las longitudes de los otros segmentos, por construccion, solo sera posible obtener dos
triangulos (uno con cada punto de interseccion de las circunferencias), que son basicamente el
mismo pero la orientacion de los angulos es contraria; as, si se sabe que dos triangulos cumplen
tener lados respectivamente iguales, por construccion, deben de ser iguales. Este es el conocido
criterio LLL de congruencia de triangulos; mas adelante se detallaran el resto de criterios, pero
a partir de este probaremos el siguiente resultado:
Teorema 3: En todo triangulo, se cumple que a lados iguales se oponen angulos iguales, y
viceversa.
Suponga que 4ABC es tal que AB = AC, entonces, por criterio LLL, 4ABC es congruente
al 4ACB (en ese orden, porque AB = AC, BC = CB y CA = BA), entonces, los angulos
que se oponen a los angulos iguales son iguales. Para el recproco necesitamos otro criterio de
congruencia, por lo que la demostracion se dejara incompleta; retome esto en la seccion de
congruencia de triangulos.
Teorema 4: En todo triangulo se cumple que a mayor lado se opone mayor angulo y viceversa.
Este teorema se deja como ejercicio para el lector (Sugerencia: utilice el teorema anterior, tome
el lado mayor y defina un punto adecuado que genere un triangulo con dos lados iguales.)

DE TRIANGULOS.

CLASIFICACION
1. Con relacion a sus lados:
8

a) Escaleno: si sus tres lados no son congruentes.


b) Isosceles: si por lo menos dos de sus lados son congruentes.
c) Equilatero: si sus tres lados son congruentes (note un triangulo equilatero es tambien
isosceles, y que los tres angulos internos son iguales entre s e iguales a 60)
2. Con relacion a sus angulos internos:
a) Acutangulo: si su angulo mayor es agudo (note que entonces los tres angulos son
agudos)
b) Rectangulo: si su angulo mayor es angulo recto (note que el angulo en cuestion es
u
nico y que los otros dos angulos son agudos; as, en un triangulo rectangulo, la
hipotenusa es mayor a los catetos)
c) Obtusangulo, si el angulo mayor es angulo obtuso (note que el angulo en cuestion
es u
nico y que los otros son agudos; as, en un triangulo obtusangulo, el lado que se
opone al angulo obtuso es el lado mayor)

LINEAS NOTABLES EN UN TRIANGULO.


1. Altura: Se llama altura de un triangulo al segmento que parte de uno de sus vertices y
llega en forma perpendicular al lado opuesto o a su prolongacion.
2. Mediana: Se llama Mediana al segmento que une un vertice con el punto medio del lado
opuesto.
3. Mediatriz: de lado de un triangulo es la recta perpendicular a dicho lado en su punto
medio.
4. Una Bisectriz: es la recta que divide en dos angulos iguales a un angulo dado; en
particular, es bisectriz interna si es la bisectriz de un angulo interno de un triangulo, y
bisectriz externa si es la bisectriz de un angulo externo de un triangulo.

DISTANCIA DE UN PUNTO A UNA RECTA.


Sea P un punto exterior a una recta L, la longitud de la perpendicular P M a la recta L es
la distancia del punto P a dicha recta. Esta perpendicular tiene la propiedad de ser u
nica y
su longitud es la distancia mnima del punto a la recta (Pruebelo utilizano el hecho que la
hipotenusa es mayor que los catetos)
Los segmentos P A y P B no son perpendiculares a L y se llaman oblicuas.

TEOREMA DE PITAGORAS.
Abordamos el estudio de las Relaciones Metricas, del cual solo realizaremos el analisis del famoso Teorema de Pitagoras, cuyo enunciado es el siguiente:
Teorema: Pit
agoras. En un triangulo rectangulo, el cuadrado de la hipotenusa es igual a la
suma de los cuadrados de los catetos.

Figura 10
Una demostracion de este teorema es debida a Thabit ibn Qurra (836-901), la cual consiste en
diseccionar la figura que se forma al construir dos cuadrados de lados respectivamente iguales
a los catetos de un triangulo rectangulo, como se muestra en el grafico 11.

Figura 11
Recproco del teorema de Pit
agoras: Si en un triangulo el cuadrado de un lado es igual a
la suma de los cuadrados de los otros dos lados, el triangulo es rectangulo.5

Ejercicios.

1. En la figura adjunta ambos triangulos son


equilateros. Encuentre el valor de .

Ver demostraci
on en la secci
on de congruencia de triangulos

10

2. En la figura 12, calcular el x si el AOB = 100 y L1 k L2 .

Figura 12
3. (*) En la figura 13, ABDE es un cuadrado y BCD es un triangulo isosceles con BD =
DC. Si ABC = 160, determinar la medida de AEC.

Figura 13

4. (*) (XV Competencia de Clubes Cabri Primera Ronda) En la figura


adjunta, ABCD es un rectangulo tal que AB = 2BC. M es el punto
medio de AB y los triangulos AM E y M BF son equilateros. Si P
es la interseccion de las rectas DE y CF , encuentre los angulos del
4CDP .

5. Si AB y F G son rectas paralelas, el ABC = CDE = , el DEF =


150. Calcule . Figura 14

y el GF H =

6. Probar que la bisectriz exterior de un triangulo es paralela al lado opuesto si y solo si el


triangulo es isosceles.
11

Figura 14
7. (*) Hallar la suma de los angulos +  + + en la figura 15.

Figura 15
8. Determine el valor de la suma A + B + I + H + F + G. Figura 16.

Figura 16

12

9. En el 4ABC el BAC = 36y AC = AB. Probar que la bisectriz interior BD, D en


AC, es congruente con el lado BC.
10. Sea ABC un triangulo rectangulo en B con AB = BC, se construye exteriormente el
triangulo equilatero BCD. Encuentre el angulo DAB.
11. En el 4ABC, AB = AC y D un punto sobre la recta AC, tal que BC = BD = DA.
Determine la medida del angulo ABD, si:
a) D esta entre A y C.
b) A esta entre D y C.
12. En un 4ABC, D es un punto sobre el lado AC tal que AB = AD. Si ABC CAB =
90, hallar el CBD.
13. En la figura 17, el ABC = ACE, DC = EC, Que lnea notable es AD del 4ABC?

Figura 17
14. Se tiene un triangulo isosceles ABC, AB = BC en el cual se traza al altura AF tal que
BF = 6 y F C = 2. Hallar AC.
15. Cual es el valor de b a en la figura 18?

Figura 18
16. La hipotenusa BC de un triangulo rectangulo ABC se divide en 4 segmentos congruentes
por los puntos G, E y H. Si BC = 20, encuentra la suma de los cuadrados de las longitudes
de los segmentos AG, AE y AH. Figura 19.
13

Figura 19
17. (*) Dado un cuadrado ABCD, se construyen los triangulos equilateros ABP (exteriormente) y ADQ (interiormente). Probar que C, P y Q estan alineados.
18. (*) Sea ABC un triangulo rectangulo con CAB = 90. D es un punto sobre la prolongacion de BC tal que BD = BA. E es un punto en el mismo semiplano que A respecto
de BC, tal que CE BC y ademas CE = CA. Mostrar que A, D y E estan alineados.
19. El cuadrilatero ABCD mostrado en la figura 20 cumple que AB k CD y BC k DA.6
Sobre las prolongaciones de AB y AD se construyen puntos E y F tales que BC = BE
y DC = DF . Demuestre que C, E y F estan alinedos.

Figura 20

20. (*) En la figura adjunta, AB = BC = CD =


DE = EF = F G = GA. Calcule la medida del
DAE.

El cuadril
atero ABCD es un paralelogramo.

14

21. Los lados de un triangulo isosceles son 12 y 5 metros, cual es su permetro?


22. Muestre que los lados de un triangulo cumplen que |a b| < c y que c <

a+b+c
.
2

23. Muestre que es posible construir un triangulo con segmentos de longitudes a, b, c si y solo
existen n
umeros positivos x, y, z tales que: a = x + y, b = y + z, c = z + x.
Problemas de Refuerzo.
24. (*) (Etapa semifinal Estatal de XXII Olimpiada Mexicana de Matematicas) En la figura
21 se muestra un hexagono regular ABCDEF de lado 1. Los arcos del crculo que estan
dibujados tienen centro en cada vertice del hexagono y radio igual a la distancia al vertice
opuesto. P , Q, R, S, T y U son los puntos de corte de estos arcos. Cuanto mide cada
lado del hexagono P QRST U ?

Figura 21
25. (*) (XXVIII Olimpiada Brasile
na de Matematica) En la figura 22, AB = AC, AM = AN
y CAM = 30, encuentre el valor del BM N .

Figura 22

15

3.

Congruencia de Tri
angulos.

CRITERIOS DE CONGRUENCIA.
Definici
on de Congruencia de tri
angulos. El 4ABC es congruente al 4A0 B 0 C 0 si: AB =
A0 B 0 , AC = A0 C 0 , BC = B 0 C 0 , ABC = A0 B 0 C 0 , ACB = A0 C 0 B 0 y BAC = B 0 A0 C 0 .
Simbolicamente: 4ABC = 4A0 B 0 C 0 . Vease figura 23.

Figura 23: Definicion de Igualdad de Triangulos.


La definicion anterior establece que dos triangulos son congruentes si tanto los lados como
los angulos se presentan en pares respectivos congruentes. Esto, seg
un la vision de Euclides,
significa que un triangulo es posible superponerlo sobre el otro (se puede desplazar, girar o
reflejar) y coincidira de manera perfecta. Sin embargo, es importante mencionar que en muy
raras ocasiones se tendra a disposicion tanta informacion, de all la importancia de los criterios
de congruencia, que establecen los requisitos mnimos para garantizar que dos triangulos son
congruentes.
El siguiente es el primero de los tres criterios de congruencia de triangulos, y se denomina

criterio de LADO-ANGULO-LADO,
en smbolos: L-A-L.
Criterio L-A-L. Si los triangulos ABC y A0 B 0 C 0 presentan las congruencias: AB = A0 B 0 ,
AC = A0 C 0 y BAC = B 0 A0 C 0 , entonces 4ABC = 4A0 B 0 C 0 .

Figura 24: Criterio LAL


Seg
un el criterio L-A-L, dos triangulos son congruentes si en uno de ellos existen dos lados y el
angulo (comprendido entre dichos lados), respectivamente congruentes a dos lados y el angulo
(comprendido entre dichos lados), en el otro triangulo.
Criterio A-L-A. Sean ABC y A0 B 0 C 0 dos triangulos tales que: AC = A0 C 0 , BCA = B 0 C 0 A0
y BAC = B 0 A0 C 0 , entonces 4ABC = 4A0 B 0 C 0 .
16

Figura 25: Criterio ALA.


Criterio L-L-L. Si un triangulo tiene sus tres lados respectivamente congruentes a los tres
lados de otro triangulo, entonces estos dos triangulos son congruentes.

Figura 26: Criterio LLL.


Ahora demostraremos el Recproco del Teorema de Pitagoras.
Demostraci
on: Sea ABC un triangulo talque BC 2 = AB 2 + AC 2 , por construccion sea el
4A0 B 0 C 0 rectangulo en A0 tal que A0 B 0 = AB y A0 C 0 = AC, entonces por el teorema de
Pitagoras B 0 C 02 = A0 B 02 + A0 C 02 , as que B 0 C 02 = BC 2 , de donde B 0 C 0 = BC y por el criterio
LLL, se deduce que el 4A0 B 0 C 0 = 4ABC, por lo tanto el BAC = B 0 A0 C 0 = 90.

TEOREMA DE LA BASE MEDIA


En todo triangulo, el segmento que une los puntos medios de dos lados es paralelo al tercer
lado e igual a su mitad.
En la figura 27, M N es el segmento que une los puntos medios de los lados AB y BC del 4ABC,
AC

a este segmento se le llama BASE MEDIA DEL TRIANGULO.


Se verifica que M N =
y
2
que M N k AC.
Demostracion:
1. Prolongar el segmento M N hasta el punto P tal que M N = N P .
2. Los triangulos M N B y P N C son congruentes, ya que BN = N C, M N = N P y el
N CP = M BN , por consiguiente, el N CP = M BN , por lo tanto, CP k M B (Por
angulos alternos internos iguales). Ademas, P C = M B = M A; con lo cual se tiene que:
M A = P C.
17

Figura 27: Teorema de La Base Media.

Figura 28: Menor Media en un Triangulo Rectangulo.


3. Uniendo el punto A con el punto P se forman los triangulos congruentes AM P y ACP
(por L A L) ya que M A = P C, AP = AP , M AP = AP C (por angulos alternos
internos entre las paralelas M A y P C). Luego, M P = AC, entonces N P = 12 M P = 12 AC.
Ademas, P AC = M P A, de donde M P k AC o que M N k AC.
Corolario: Menor mediana de un tri
angulo rect
angulo. En todo triangulo rectangulo,
la mediana relativa a la hipotenusa es la mitad de la longitud de la hipotenusa y es la menor
de las tres medianas del triangulo.
Demostraci
on: En la figura 28, BM es la mediana relativa a la hipotenusa AC del 4ABC,
; (con lo cual se tendra que BM = AM = M C). Si por M se traza
probaremos que BM = AC
2
una paralela al lado AB, que corte al lado BC en N , entonces N es el punto medio de BC y
el M N C = 90, los triangulos BN M y CN M son congruentes por el criterio L-A-L, luego
M B = M C = AM .
Probar que BM es la menor mediana (Ejercicio).

18

Ejercicios.

1. (*) En la figura adjunta, ABC es un triangulo equilatero


y CDEF es un cuadrado. Se construye un punto G tal
que CF = CG y ademas CF G = 15. Probar que
AGC = BDC.

2. Dado un triangulo equilatero ABC, se construye un triangulo equilatero DEF cuyos


vertices estan sobre los lados del 4ABC, tal como muestra la figura 29. Demuestre que
los triangulos ADF , BED, CF E son todos congruentes entre si.

Figura 29
3. ABCD es un cuadrado, E, F , G y H son puntos sobre los lados AB, BC, CD, DA, respectivamente, tal que EF GH tambien es cuadrado. Demuestre que los triangulos AEH,
BF E, CGF , DHG son todos congruentes entre si. Figura 30.

Figura 30

19

4. ABCDE y F GHIJ son pentagonos regulares (Vease figura 31). Demuestre que los
triangulos AF J, BGF , CHG, DIH, EJI son todos congruentes entre si.

Figura 31
5. Demuestre que dos triangulos desplazados son congruentes.
6. Demuestre que dos triangulos rotados son congruentes.
7. Demuestre que dos triangulos reflejados con respecto a un punto

son congruentes.

8. Demuestre que dos triangulos reflejados con respecto a una recta son congruentes.
Importante: Las traslaciones, rotaciones y reflexiones no cambian el tama
no ni la forma
de un triangulo.
9. (*) (Cuaderno de Olimpiadas Mexicanas - Geometra) En la figura 32, ABCD un cuadrado y EF GH. Demuestre que que EF = GH.

Figura 32
10. Dos cuadrados ABCD y EHGF , ambos de lado l, estan colocados en manera tal que un
vertice de uno esta en el centro del otro (vease la figura 33). Demuestre que el area del
l2
cuadrilatero EJBK es
y por ende no depende de la posicion de J (o K).
4
7

La reflexi
on con respecto a un punto es equivalente a una rotacion de 180

20

Figura 33
11. (*) Sea ABCD un cuadrado. Se construyen triangulos equilateros ADP y ABQ como se
muestra en la figura 34. Sea M la interseccion de CQ con AD y N la interseccion de CP
con AB. Demuestre que CM N es un triangulo equilatero.

Figura 34
12. En un 4ABC el B = 2C, la mediatriz del lado AC corta en F al lado BC. Hallar
AB, si BF = 3 y F C = 9.
13. (*) (Examen final de XVI Olimpiada mexicana de Matematica) Los angulos de un triangulo ABC estan en progresion aritmetica (B A = C B = ), D, E, y F son los
puntos medios de los lados BC, CA y AB, respectivamente. Llamamos H al pie de la
altura trazada desde C (que cae entre B y F ) y G a la interseccion entre DH y EF .
Cuanto vale F GH?

21

14. En la figura 35, AC = 12 AF = 4 y BAF = 30. Hallar BF si AG = GC.

Figura 35

15. En la figura adjunta AG = GC, el AF G = 20. Hallar el


F AC, si AC = 2BF .

Problemas de Refuerzo.
16. En la figura 36, ABC, CDE y EF A son triangulos isosceles, con el ABC = CDE =
EF A = 120. Probar que el 4BDF es equilatero.

Figura 36
17. (*) 4ABC es un triangulo isosceles con ABC = ACB = 80. D es un punto en AC
tal que ABD = 10. Demuestre que AD = BC.

22

4.

Cuadril
ateros: Clasificaci
on y Propiedades.

CLASIFICACION.
Los cuadrilateros pueden clasificarse de acuerdo a sus diagonales de la siguiente forma:
Cuadril
atero Convexo: Es un cuadrilatero con las dos diagonales en su interior.
Cuadril
atero Entrante: Es un cuadrilatero con una diagonal en el interior y otra en el exterior.
Cuadril
atero Cruzado Es un cuadrilatero con las diagonales en su exterior.8
Es muy frecuente que se considere que un cuadrilatero es convexo, a menos que se especifique lo
contrario. Esto es as porque muchos resultados son mas claros en un cuadrilatero convexo, sin
embargo, es importante darse cuenta que existen teoremas que no se cumplen para cualquier
tipo de cuadrilateros, por ejemplo:
Teorema: La suma de los angulos internos de un cuadrilatero no cruzado es 360.
La demostracion de este resultado se basa en la diseccion del cuadrilatero en dos triangulos cuyos angulos internos conforman los angulos internos del cuadrilatero, sin embargo, estas
condiciones no pueden lograrse en un cuadrilatero cruzado; de hecho, la suma de los angulos
internos puede hacerse arbitrariamente peque
na cuando el cuadrilatero es cruzado.
Tambien hay otras clasificaciones de cuadrilateros de acuerdo a sus lados y angulos.
Cuadril
atero Equi
angulo: un cuadrilatero (convexo) es equiangulo si todos sus angulos internos son iguales; dado el teorema anterior, los angulos son iguales a 90, por ello este cuadrilatero
es llamado rectangulo.
Cuadril
atero Equil
atero: un cuadrilatero (convexo) es equilatero si todos sus lados son iguales. A este cuadiratero tambien se le conoce como rombo.
Cuadrado: es un cuadrilatero que es equiangulo y equilatero.
Paralelogramo: es un cuadrilatero con los lados opuestos paralelos.
Trapecio: es un cuadrilatero con un par de lados opuestos paralelos.9
8

Tanto los cuadril


ateros convexos como los entrantes son cuadrilateros simples, que son los cuadril
ateros
cuyos lados no se cortan salvo en los extrenos; en contraposicion, los cuadrilateros cruzados no son simples.
9
Note que un paralelogramo es tambien un trapecio.

23

PARALELOGRAMOS
Dado el paralogramo ABCD, por propiedades de angulos entre paralelas es posible probar el
siguiente resultado:
Teorema: Los angulos opuestos son iguales y los angulos consecutivos son suplementarios:
ABC = CDA = y BCD = DAB = 180 .
Por otra parte, por criterio ALA, 4ABC 4CDA; esto implica que AB = CD y BC = DA,
i.e.
Teorema: Los lados opuestos de un paralogramos son iguales.
A partir de esto, si M es la interseccion de AC con BD, por criterio ALA, 4ABM 4CDM ,
por lo que AM = CM y BM = DM , i.e.
Teorema: Las diagonales de un paralelogramo se bisecan.
Ademas, se cumple un resultado sofisticado y muy importante:
Teorema: Ley del Paralelogramo. Si ABCD es un paralelogramo entonces el doble de la
suma de los cuadrados de los lados es igual a la suma de los cuadrados de las diagonales, es
decir

2 AB 2 + BC 2 = AC 2 + BD2
Demostraci
on: Aplicando la Ley del Coseno a 4ABC y 4ABD se tiene
AC 2 = AB 2 + BC 2 AB BC cos
DB 2 = AB 2 + AD2 AB AD cos(180 )

AC 2 + DB 2 = 2 AB 2 + BC 2 AB BC (cos + cos(180 ))
y dado que cos = cos(180 ) el resultado se sigue inmediatamente.

RECTANGULOS
En primer lugar, es importante notar que todo rectangulo es paralelogramo (por angulos entre paralelas), por lo que todos los resultados probados anteriormente son heredados a todo
rectangulo; pero los rectangulos tienen propiedades adicionales:
Observe que por criterio LAL, 4ABC 4ABD, por lo que AC = BD y entonces
Teorema: Las diagonales de un paralelogramo son iguales; ademas, el punto de interseccion
de estas equidista de los cuatro vertices y por tanto es el centro de una circunferencia que pasa
por todos los vertices.
Por otra parte, observe que si se aplica la ley del paralelogramo a un rectangulo se obtiene el
Teorema de Pitagoras.
24

ROMBOS
Dado un rombo ABCD, por criterio LLL, 4ABC 4CDA, y por lo tanto BAC = DAC
y BCA = DAC, lo cual implica BC k AD y AB k CD, i.e., todo rombo ABCD es un
paralelogramo. Ademas, por las mismas congruencias se tiene
Teorema: Las diagonales de un rombo cumplen ser una mediatriz de la otra.
Teorema: Las diagonales de un rombo bisecan a los angulos interiores del rombo; esto implica
que el punto de corte de las diagonales equidista de los cuatro lados del rombo y es el centro
de una circunferencia tangente a estos.

TRAPECIOS
Dado el trapecio ABCD (con AB k CD), se construyen los puntos medios de BC y DA, M y
N , respectivamente. Si el cuadrilatero M N AB se rota con centro en M y angulo 180 se genera
un cuadrilatero M N 0 A0 C; observe que N D = N 0 A0 y N D k N 0 A0 , por lo que DN N 0 A0 es un
paralelogramo y
N N 0 = DA0
2M N = DC + CA0
2M N = DC + AB
AB + CD
MN =
2
El segmento M N es llamado base media del trapecio, y por lo recien demostrado se tiene
Teorema: La base media de un trapecio es igual a la semisuma de las bases.
Por otra parte, hay ciertos trapecios que reciben nombres particulares; el trapecio rectangulo es
aquel que las bases son perpendiculares a alguno de los otros lados; y por otra parte, el trapecio
isosceles es aquel que los lados (distintos de las bases) tienen igual longitud. 10

Ejercicios
1. Dado el trapecio ABCD con AB k CD, demuestre que la bisectriz interior del A es
paralela a la bisectriz exterior del D.
2. A un rombo ABCD se le construyen exteriormente los cuadrados ABEF y BCGH.
Demuestre que 4ABD = 4EBH.
3. (*) Sea ABCD un paralelogramo. Se construyen triangulos equilateros exteriores 4CDP
y 4ADQ. Demuestre que el 4BP Q es equilatero.
4. Si un cuadrilatero tiene dos lados iguales y paralelos es un paralelogramo.
10

Los trapecios is
osceles son muy importantes cuando se estudian los angulos en la circunferencia; resulta que
un trapecio es is
osceles si y s
olo si los cuatro vertices se ubican sobre una misma circunferencia.

25

5. Demuestre que las bisectrices interiores de un paralelogramo forman un rectangulo (que sucede si el paralelogramo es ademas rombo?).
6. Demuestre que las bisectrices exteriores de un paralelogramo forman un rectangulo.
7. Sea ABCD un paralelogramo. La bisectriz interna del CDA corta a BA en M , y la
bisectriz interna del BAD corta a CD en N . Demuestre que ADN M es un rombo.
8. Demuestre que si por el punto de interseccion de las diagonales de un rombo se trazan perpendiculares a los lados del rombo, entonces los puntos de interseccion de dichas
perpendiculares con los lados del rombo forman un rectangulo.
9. Demuestre que las bisectrices de los angulos definidos por las diagonales de un rombo,
cortan a los lados del rombo en cuatro puntos que forman un cuadrado.
10. En un 4ABC sea G la interseccion de las medianas BB 0 y CC 0 . Sean B 00 , C 00 las reflexiones
de G respectivas a los puntos B 0 y C 0 .
a) Demuestre que AGCB 00 y AGBC 00 son paralelogramos.
b) A partir de lo anterior, demuestre que BCB 00 C 00 tambien es paralelogramo.
c) Demuestre que A0 pertenece a la recta AG, y concluya que las tres medianas de un
triangulo concurren en el punto G, llamado el centroide del 4ABC.
d) Demuestre que CG = 2GC 0 ; relaciones similares se cumplen para las otras dos medianas.
11. Teorema de Varignon: Dado un cuadrilatero ABCD (no necesariamente convexo), se
construyen los puntos medios L, M , N , O, P , Q, de los segmentos de recta AB, BC, CD,
DA, BD, AC, respectivamente. Figura 37.
a) Demuestre que LM N O, LP N Q, OP M Q, son paralelogramos.
b) Demuestre que LN , OM , P Q concurren en un punto, llamado el centroide del cuadrilatero ABCD.
c) Demuestre que el permetro de LM N O es igual a AC + BD; resultados similares se
cumplen para los otros paralelogramos.
12. Sea ABCD un paralelogramo tal que existe un punto E sobre el lado AB que cumple
CED = 90. Sean M y N los pies de las perpendiculares trazadas desde A y B hacia
DE y CE, respectivamente. Demuestre que AC, BD y M N concurren.
13. (*) (Hector Alberti) Sea ABCD un cuadrado. Se construyen los triangulos equilateros
BDA0 , ACB 0 , BDC 0 y ACD0 . Demuestre que el A0 B 0 C 0 D0 es tambien un cuadrado.

26

Figura 37: Teorema de Varignon


14. (*) (II Olimpiada Matematica del Cono Sur) En la figura 38 ABCD y AECF son paralelogramos. Demuestre que BEDF es paralelogramo.

Figura 38
Problemas de Refuerzo.
15. (*) ABCD es un cuadrilatero convexo y O es un punto en su interior. Sean P , Q, R, S,
los puntos medios de los lados AB, BC, CD, DA, respectivamente. Por P se traza una
paralela a OR, por Q se traza una paralela a OS, por R se traza una paralela a OP , y
por S se traza una paralela a OQ. Demuestre que estas cuatro rectas concurren.
16. (*) Un trapecio isosceles tiene diagonales perpendiculares y su area es 2010, determine su
altura.
17. (*) (IX Competencia de Clubes Cabri, Segunda Ronda) Sea ABCDEF un hexagono
regular cuyo centro es O. Se construyen los cuadrados F SOP y ORCQ. Demuestre que
AP QB y SEDR son rectangulos. Figura 39.

27

Figura 39
18. (*) Sobre los lados del 4ABC se trazan exteriormente los cuadrados ABP Q, CARS y
BCT U . Luego se trazan los paralelogramos AQA0 R, CSC 0 T y BU B 0 P .
a) Sean A00 , B 00 , C 00 los centros de los cuadrados BCT U , CARS, ABP Q, respectivamente. Demuestre que estos centros estan sobre los lados del 4A0 B 0 C 0 .
b) Demuestre que AA00 , BB 00 , CC 00 concurren.
19. (*) Se dibujan cuadrados exteriores a los lados de un paralelogramo, demuestre que:
a) El cuadrilatero determinado por los centros de esos cuadrados es un cuadrado.
b) Las diagonales de ese cuadrado son concurrentes con las del paralelogramo.
20. (*) Dado un 4ABC, se construyen exteriormente los triangulos rectangulo isosceles
4ACP y 4BCQ, con AC y BC como hipotenusas. Si M es el punto medio de AB,
demuestre que el 4M P Q tambien es un triangulo rectangulo isosceles.

28

5.

Angulos
en la Circunferencia.

LA CIRCUNFERENCIA Y SUS ELEMENTOS


Una circunferencia es el lugar geometrico de puntos que equidistan de un punto dado, llamado
el centro de la circunferencia; la distancia de cada punto de la circunferencia al centro es el radio.
Por otra parte, todos los puntos que estan a una distancia del centro menor o igual al radio
forman el crculo; estos puntos quedan al interior o sobre la circunferencia.
Si A y B son dos puntos de una circunferencia, el segmento de recta AB define una cuerda; en
particular, si el centro de la circunferencia pertenece a la cuerda, esta es llamada diametro. Es
importante mencionar que para cada punto de la circunferencia existe exactamente un punto
diametralmente opuesto.
En la figura 40, se tiene una circunferencia de centro O y radio r = OA = OB = OA0 ; AB
y AA0 son cuerdas, pero AA0 es tambien diametro, i.e, A0 es diametralmente opuesto a A y
viceversa. Observe que por la desigualdad triangular aplicada al triangulo isosceles 4AOB

Figura 40

AB < AO + BO
= r+r
= AA0
Si A es un punto fijo, esta desigualdad es valida para cualquier punto B sobre la circunferencia
(excepto cuando B = A0 lo cual implica AB = AA0 ). Esto quiere decir que el diametro es la
mayor de todas las cuerdas.
A las porciones de circunferencia que quedan entre dos puntos ubicados en la circunferencia,
se les llama arcos de circunferencia; note que dos puntos sobre una circunferencia definen dos
arcos de circunferencia. Tambien, si un angulo tiene vertice sobre el centro de la circunferencia y
esta formado por dos radios, sera llamado angulo central ; de nuevo, AOB hace referencia a dos
angulos, cuya suma es 360, y subtienden respectivamente a uno de los arcos AB. Finalmente,
si un angulo tiene el vertice sobre la circunferencia y esta formado por dos cuerdas, sera llamado angulo inscrito; en la figura anterior, AA0 B es un angulo inscrito que subtiende al arco AB.

29

Teorema: El angulo central es el doble del angulo inscrito que subtiende el mismo arco.
Demostraci
on: Considere la figura 41, se demostrara que AOB = 2AP B en los tres casos
mostrados. En la circunferencia de la izquierda, sea P 0 el punto diametralmente opuesto a P ;
observe que 4AP O y 4BP O son triangulos isosceles, y por el teorema del angulo externo se
tiene
AOB =
=
=
=
=

AOP 0 + BOP 0
(AP O + OAP ) + (BP O + OBP )
2AP O + 2BP O
2 (AP O + BP O)
2AP B

Figura 41
El caso de la circunferencia del medio es mas sencillo y se deja como ejercicio para el lector.
Para la circunferencia de la derecha, el trabajo es analogo y solo cambia en un peque
no arreglo
algebraico
AOB =
=
=
=
=

BOP 0 AOP 0
(BP O + OBP ) (AP O + OAP )
2BP O 2AP O
2 (BP O AP O)
2AP B

Corolario: Todos los angulos inscritos que subtienden el mismo arco son iguales (Ver figura
42). En particular, los angulos internos son iguales a 90 si subtienden a una semicircunferencia.
Demostraci
on: Todos los angulos mostrados en la figura 42 son iguales a la mitad del AOB,
y por tanto, son iguales entre s. En particular, si AB fuera un diametro, AOB = 180 y por
tanto AP B = 90. 11
Hay un par de angulos mas que son importantes: Si un punto P es interno a la circunferencia,
el angulo de vertice P formado por dos cuerdas que pasan por P se llama angulo interior. De
11

Observe que en cualquier tri


angulo rect
angulo, el punto medio de la hipotenusa equidista de los tres vertices.

30

Figura 42
forma similar, si P es exterior y dos cuerdas de la circunferencia (al prolongarse) pasan por P ,
el angulo con vertice P es llamado angulo exterior.
Dejamos como ejercicio demostrar el siguiente teorema:
Teorema: Los angulos interior y exterior mostrados en la figura 43 cumplen las formulas
siguientes:
BOD + AOC
2
BOD AOC
AP C =
2

AQC =

Figura 43

CUADRILATEROS
CICLICOS
Ahora suponga que sobre una circunferencia se ubican cuatro puntos A, B, C, D, como se
muestra en la figura 44. Al cuadrilatero ABCD se le llama cuadrilatero cclico o concclico.
Observe que

ABC + CDA = + = 180 .
2
2
31

Figura 44
Y analogamente DAB +BCD = 180. Esto significa que si ABCD es un cuadrilatero cclico
y convexo, entonces los angulos opuestos son suplementarios. Tambien, es posible demostrar por
contradiccion el recproco de este resultado: si suponemos que ABCD es tal que B+D = 180
pero no es cclico, se define el punto D0 como la otra interseccion de AD con el circuncrculo
del 4ABC, y como ABCD0 es cclico (por construccion) entonces B + D0 = 180, luego,
D = D0 , lo cual implica la contradiccion CD k CD0 (rectas paralelas que se cortan en C).
As, se ha demostrado el siguiente teorema:
Teorema: El cuadrilatero convexo ABCD es un cuadrilatero cclico si y solo si
A + C = 180 = B + D
Tambien, otro criterio muy u
til y cuya demostracion tambien se basa en el corolario anterior es
Teorema: El cuadrilatero convexo ABCD es un cuadrilatero cclico si y solo si se cumple
alguna de las siguientes igualdades
ABD
BCA
BAC
CAD

=
=
=
=

ACD
BDA
BDC
CBD

Es importante recalcar que NO todo cuadrilatero puede ser inscrito en una circunferencia; por
ejemplo, un paralelogramo no sera cclico a menos que sea rectangulo.

RECTAS Y CIRCUNFERENCIAS TANGENTES A UNA CIRCUNFERENCIA


Dada una circunferencia, una recta puede ser tangente o secante a la circunferencia, dependiendo si la corta en uno o dos puntos, respectivamente; en cualquier otro caso, se dice que la

32

recta no corta a la circunferencia.12


Sea l una recta secante a la circunferencia que corta a la circunferencia en A y B (A 6= B);
como el 4AOB es isosceles, OAB < 90. Recprocamente, si por A se traza una recta l tal
que uno de los angulos que forma con OA es menor que 90, se puede construir un punto B
sobre l tal que OAB = ABO < 90 y A 6= B (basta proyectar O sobre l y luego reflejar A
con respecto a este punto, el resultante es el punto B); entonces el 4AOB es isosceles, por lo
que OA = r = OB, i.e. B pertenece a la circunferencia y por tanto l corta a la circunferencia
en dos puntos distintos. As
Teorema: Una recta l corta a una circunferencia de centro O en dos puntos distintos A y B si
y solo si un angulo entre l y OA es agudo.
Corolario: Si l es una recta tangente en A a una circunferencia de centro O, ninguno de los
angulos entre l y OA puede ser agudo, y por tanto l OA.
A partir de este resultado se prueban otros resultados muy conocidos y u
tiles, que dejamos de
ejercicios para el lector.
Teorema: Dado un punto P externo a una circunferencia de centro O, si P A y P B son segmentos tangentes a la circunferencia en A y B, respectivamente, entonces el cuadrilatero P AOB es
cclico y bisosceles.
Corolario: Dado un punto P externo a una circunferencia de centro O, la circunferencia de
diametro P O corta a la circunferencia dada en dos puntos A y B tales que P A y P B son rectas
tangentes.
Definici
on: El angulo semi-inscrito en una circunferencia es aquel que se forma con una cuerda
y la recta tangente en alguno de los extremos de la cuerda.
Teorema: La media del angulo semi-inscrito definido por la cuerda AB es igual a la medida
de un angulo inscrito que subtiende al arco AB.
Demostraci
on: Considere la figura 45. Como AP BO es cclico, entonces P AB = P OB;
ademas, como P O es la mediatiz de AB, P OB = P OA, por lo que
P AB =

AOB
= AQB
2

Por otra parte, dada una circunferencia, otra circunferencia puede ser secante o tangente a la
primera, dependiendo si la corta en uno o dos puntos, respectivamente; en cualquier otro caso
se dice que las circunferencias no se cortan.13
12

Cuando la recta es tangente a la circunferencia puede considerarse como un caso muy peculiar en el cual
los dos puntos de corte coinciden.
13
Tambien ac
a puede considerarse a las circunferencias tangentes como un caso especial de circunferencias
secantes en el cual los puntos de corte coinciden.

33

Figura 45
Ademas, dos circunferencias pueden posicionarse una dentro de la otra, y claramente, la circunferencia de radio mayor es la externa mientras que otra es la interna; particularmente, si las
circunferencias tienen el mismo centro se llaman concentricas. Finalmente, combinando estas
definciones se tienen las circunferencias tangentes exteriormente y las tangentes interiormente.
Teorema: Dadas dos circunferencias de centros O1 y O2 que se cortan en dos puntos distintos
A y B, se cumple que O1 O2 AB.
Teorema: Si dos circunferencias de centros O1 y O2 son tangentes en A, se cumple que O1 , A
y O2 estan alineados.
Teorema:
a) Dos circunferencias, una dentro de la otra, no tienen rectas tangentes en com
un.
b) Dos circunferencias tangentes interiormente tienen una recta tangente com
un.
c) Dos circunferencias secantes (en dos puntos distintos) tienen dos rectas tangentes en com
un.
d) Dos circunferencias tangentes exteriormente tienen tres rectas tangentes en com
un.
e) Dos circunferencias no secantes y tal que ninguna contiene a la otra, tienen cuatro rectas
tangentes en com
un.

Ejercicios

1. Si el M P Q = 20, determine el valor del QON en la figura


adjunta.

34

2. Dado un angulo inscrito BAC, y su angulo central BOC, se sabe que BAC + BOC =
180. Calcular el OBC.
3. En la figura 46, BCDO es un rombo. Determine el valor del angulo y la medida de las
diagonales de BCDO si el radio de la circunferencia mide 6.

Figura 46
4. Un cuadrilatero cclico ABCD satisface ABC = 2CDA = . Calcule .
5. En la figura 47, P R es una tangente com
un. Calcule el valor del P QR.

Figura 47

6. En la figura adjunta, el AF E = 100y el BCD =


150. Calcule el AGB.

7. Dado un angulo AOB, se trazan dos rectas l OA y m OB. Si P es el punto de


corte de l y m, demuestre que A, B, O, P se ubican sobre una misma circunferencia.
35

8. Las bisectrices BP y CQ del 4ABC se cortan en I. Demuestre que si BAC = 60


entonces 4P QI es isosceles.
9. En la figura 48 se ha tomado un punto C sobre la circunferencia; AC y BC cortan a la
segunda circunferencia en D y E respectivamente. Probar que OC DE.

Figura 48
10. (*) Dada la figura 49, demuestre que AB k A0 B 0 .

Figura 49
11. En la figura 50 CR es una recta tangente en C, demuestre que AB k CR.

Figura 50

36

12. Dos circunferencias 1 y 2 son tangentes (interior o exteriormente) en P (Ver figura 51).
Dos rectas que pasan por P cortan a 1 y 2 en A y C, y en B y D, respectivamente.
Demuestre que AB k CD.

Figura 51

13. (*) Dos circunferencias de centros O1 y O2 son tangentes (interna o externamente) en un


punto P ; por este punto se traza una recta que corta nuevamente a la circunferencias en
A y B, respectivamente. Demuestre que AO1 k BO2 .
14. Dos circunferencias son tangentes externamente en el punto A. Una tangente exterior
com
un toca a una circunferencia en B y a la otra en C. Demostrar que BAC = 90.
15. En la figura 52, DE es tangente en D, y C es el punto medio del arco AD. Encuentre el
valor del angulo seminscrito ADE.

Figura 52

37

16. Determine el valor del DCF , sabiendo BE es tangente en el punto D a la circunferencia


de centro O. Ver Figura 53.

Figura 53
17. Si el AEB = 30, ADE = 20 y ACE = 35, calcule el AF B. Vease figura 54.

Figura 54
18. Dada una circunferencia de diametro BC, se toma un punto P en la prolongacion de BC,
y se traza la tangente AP . Si AP = AB y O es el centro de la circunferencia, demuestre
que el 4AOC es equilatero.
19. (*) Dadas dos circunferencias una fuera de la otra, demuestre que las tangentes comunes
externas forman segmentos iguales; analogamente, las tangentes comunes internas forman
segmentos iguales.
20. (*) Teorema de Pithot. Demuestre que en todo cuadrilatero inscribible, la suma de
lados opuestos es igual.
21. (*) Teorema de Steiner. En todo cuadrilatero exinscrito a una circunferencia, la diferencia de las longitudes de lados opuestos es igual.
38

22. Demuestre que las mediatrices de un cuadrilatero son concurrentes si y solo si es cclico.
23. Demuestre que el cuadrilatero convexo ABCD es inscribible si y solo si los incrculos
respectivos del 4ABC y 4CDA son tangentes.
24. Demuestre que las bisectrices internas de un cuadrilatero son concurrentes si y solo si es
inscribible.
25. Demuestre que todo rombo es inscribible.
26. En la figura 55, AB es una cuerda y por D se traza una recta tangente a la circunferencia
paralela a AB. Demuestre que CD es bisectriz del ACB.

Figura 55
27. Determine las medidas de ACB y ACO de la figura 56.

Figura 56

28. Cuatro cilindros de diametro 1 estan pegados apretadamente por


una cuerda muy fina, como en la figura adjunta. Demostrar que la
cuerda tine longitud 4 + . Demostrar tambien que el area sombreada entre los cilindros es 1 4 .

29. En la figura 57, ABCD es un trapecio isosceles con AB k CD y DA = BC = 2; tomando


DA y BC como diametros, se construyen dos circunferencias tangentes. Si DC = 3AB,
calcule el area del trapecio.
39

Figura 57
30. La figura 58 esta formada por un paralelogramo y dos circunferencia tangentes entre s y
tangentes a tres lados del paralelogramo. Sabiendo que el radio de las mismas mide la
cuarta parte del lado menor del paralelogramo, calcule la razon entre el lado mayor del
paralelogramo y el radio de las circunferencias.

Figura 58
31. En la figura 59, ABCDEF es un hexagono regular y las circunferencias de centro en los
vertices son tangentes dos a dos. Si las circunferencias sobre los vertices B, D, F son
iguales, demuestre que las circunferencias restantes son iguales.

Figura 59
32. Alrededor de una circunferencia se construyen diez circunferencias tangentes a la original
y tangentes entre s (Vease figura 60). Demuestre que la suma de las areas de las diez
circunferencias es el doble del area de la circunferencia mayor.
40

Figura 60
33. (*) Teorema de Miquel: Dado un 4ABC, sean X, Y , Z puntos sobre AB, BC, CA,
respectivamente. Demuestre que los circuncrculos de 4AXZ, 4BY X, 4CZY tienen un
punto en com
un M .
34. (X OMCC - P2, Aaron) Sea ABCD un cuadrilatero concclico con diametro AC, y sea
O el centro de su circunferencia. Se construyen los paralelogramos DAOE y BCOF .
Demuestre que si E y F estan sobre la circunferencia entonces ABCD es rectangulo.
35. (*) Sea ABC un triangulo, y sean L y N las intersecciones de la bisectriz del angulo A
con el lado BC y el circuncrculo de ABC respectivamente. Construimos la interseccion
M del circuncrculo de ABL con el segmento AC. Prueba que los triangulos BM N y
BM C tienen la misma area.
36. (*) Sea AB el diametro de una semicircunferencia. Se colocan los puntos M y K sobre
la semicircunferencia y sobre AB, respectivamente.14 Sea P el centro de la circunferencia
que pasa por A, K y M ; sea Q el centro de la circunferencia que pasa por B, K y M .
Demuestre que M P KQ es concclico.
37. (*) Las circunferencias 1 y 2 se cortan en los puntos A y B. Por el punto A se traza
una recta que corta nuevamente a las circunferencias 1 y 2 en los puntos C y D,
respectivamente. Por los puntos C y D se trazan tangentes a las circunferencias, las
cuales se cortan en el punto M . Demuestra que M CBD es cclico.
38. (*) El 4ABC cumple que A = 90 y AB = AC. Se toma un punto E del segmento
AB, se construye interiormente un triangulo equilatero AEF . EF corta BC en I, y se
construye exteriormente un triangulo equilatero BIJ. Encuentre EJB.
39. (*) En la figura 61, se sabe que AO1 B AO2 B = 70 y ademas la tangente EB forma
el triangulo isosceles ABE, con AB = AE. Encuentre EBC.
14

M y K son distintos de A y B.

41

Figura 61
40. (*) Dos circunferencias 1 y 2 se cortan en A y B. Una recta por A corta a 1 y 2 en C
y D, respectivamente, y la paralela a CD por B corta 1 y 2 en E y F , respectivamente.
Demuestre que 4CDB 4EAF .
41. (*) La Recta de Simson-Wallace. Sean X, Y y Z los pies de las alturas trazadas
desde un punto P en el circuncrculo del 4ABC hacia AB, BC y CA, respectivamente.
Demuestre que X, Y y Z estan alineados.
42. (*) Sea P un punto exterior al cuadrado ABCD tal que AP C = 90 , Q es la interseccion
de AB y P C, y R el pie de la perpendicular por Q a CA. Demuestre que P , R y D estan
alineados.
43. En la figura 62, ABCD es un trapecio rectangulo tal que la circunferencia de diametro
AB (y centro O) es tangente a CD. Demostrar que O pertenece a la circunferencia de
diametro CD y que esta circunferencia es tangente a BA.

Figura 62
44. El 4ABC es rectangulo en C, la circunferencia de centro O es tangente a cada uno de los
lados del 4ABC en los puntos P , Q y R (como se muestra en la figura 63), y se cumple
que AP = 20 y BP = 6. Calcule OP .
42

Figura 63
45. En la figura 64 se muestran tres semicircunferencias, una de diametro AB (de centro O
y radio r), otra de diametro AO y la u
ltima de diametro OB. Determine la razon entre
el radio de la circunferencia tangente a estas tres semicircunferencias y r.

Figura 64
46. El segmento AB es diametro de un semicrculo con centro en O. Un crculo con centro
en P es tangente a AB en O y tambien al semicrculo. Otro crculo con centro en Q es
tangente a AB, al semicrculo y al crculo de centro en P . Si AB = 2, cual es el radio
del crculo con centro en Q?

Figura 65
Problemas de Refuerzo.
47. Los vertices A y B de un triangulo equilatero 4ABC estan sobre una circunferencia de
radio 1 y el vertice C esta en el interior de la circunferencia. Un punto D (distinto de B)
que esta en la circunferencia es tal que AD = AB. La recta DC corta por segunda vez a
la circunferencia en E. Encuentre la longitud del segmento CE. Ver figura 66.
48. (*) (OIM 2002, P-4) En un triangulo escaleno ABC se traza la bisectriz interior BD, con
D sobre AC. Sean E y F puntos sobre la recta BD tales que (AE k CF ) BD, y sea
M el punto sobre el lado BC tal que DM BC. Demuestre que EM D = DM F .
43

Figura 66
49. (*) (OMCC 2003, P-2) Sea S una circunferencia y AB un diametro de ella. Sea t la recta
tangente a S en B y considere dos puntos C y D en t tales que B este entre C y D. Sean
E y F las intersecciones de S con AC y AD y sean G y H las intersecciones de S con
CF y DE. Demuestre que AH = AG.
50. (*) (The 59th Romanian Mathematical Olympiad District Round) Considere un cuadrado
ABCD y un punto E sobre el lado AB. La diagonal AC corta al segmento DE en el punto
P . La perpendicular por P a DE corta al lado BC en F . Probar que EF = AE + CF .
51. (*) Teorema de Arqumedes: En la figura 67, la region delimitada por tres semicircunferencias mutuamente tangentes, es conocida como cuchilla de zapatero o arbelos.
Demostrar que las circunferencias sombreadas son congruentes.

Figura 67: Teorema de Arqumedes.

44

6.

Teorema de Thales y su recproco. Semejanza de Tri


angulos.

Introducci
on.
Definicion
1. Razon: se llama razon, al cociente de dos cantidades, expresadas en la misma magnitud,
por ejemplo ab .
2. Proporcion: se llama proporcion a la igualdad de dos razones. Por ejemplo ab = dc , 15 a los
terminos a y d se les llama extremos y los terminos b y c se les llama medios, al termino
d se le llama cuarta proporcional entre a, b y c en este orden.
Propiedades de las proporciones:
a
c
1. = si y solo si a c = b d.
b
d
2.

a
c
b
d a
b
= si y solo si = o = .
b
d
a
c c
d

3.

a
c
ab
cd
= si y solo si
=
.
b
d
b
d

4.

a
c
a+b
c+d
= si y solo si
=
.
b
d
ab
cd

Paralelismo y proporcionalidad.
Definicion
1. Un punto P AB divide al segmento AB en una razon dada r, si

PA
PB

= r.

Figura 68
2. Sean AB y CD dos segmentos, y sean P AB y Q CD, decimos que P y Q dividen a
QA
A
AB y CD en segementos proporcionales si PP B
= QB
.

Figura 69
15

En algunos textos de geometra se utiliza la notacion de proporcion as a : b :: c : d que se lee a es a b como


c es a d.

45

Teorema de Thales. Si tres paralelas cortan a dos secantes entonces los segmentos que determinan en ellas son proporcionales. 16
Antes de demostrar el Teorema de Thales, se enunciaran dos teoremas que a pesar de su apa
rente sencillez es de mucha utilidad en problemas que involucran Areas
y Proporcionalidad.
Lema 1. Sea AB k CD. Demuestre que: (ABC) = (ABD).
Lema 2. Sea P un punto sobre el lado AB (o su prolongacion) del 4ABC. Pruebe que:
AP
(AP C)
=
PB
(P BC)
.
A continuacion se enuncian los pasos a seguir en la demostracion del teorema de Thales.
Demostraci
on. Sean AA0 , BB 0 y CC 0 rectas paralelas que cortan a dos secantes en los puntos
0
0
A, A , B, B , C, C 0 respectivamente (ver figura 70).

Figura 70: Teorema de Thales


Pruebe que:
1.

AB
(ABB 0 )
=
BC
(BCB 0 )

A0 B 0
(A0 B 0 B)
2. 0 0 =
.
BC
(B 0 C 0 B)
3. (ABB 0 ) = (A0 B 0 B) y (BCB 0 ) = (B 0 C 0 B).
Con ayuda de las igualdades demostradas concluya que:
AB
A0 B 0
= 0 0.
BC
BC
Observaci
on Importante: Utilice las propiedades de las proporciones para demostrar las
equivalencias siguientes (interpretelas geometricamente):
A0 B 0
AC
A0 C 0
AC
A0 C 0
AB
= 0 0
= 0 0
= 0 0
BC
BC
AB
AB
BC
BC
16

El teorema de Thales puede enunciarse de manera general como sigue: Si tres o mas paralelas cortan a dos
o m
as secantes entonces los segmentos que determinan en ellas son proporcionales.

46

Corolario (Teorema de Thales en el triangulo). Toda recta paralela a un lado de un triangulo


y que corte a los otros dos lados, divide a estos lados en segmentos proporcionales.
Recproco del Teorema de Thales. Si tres rectas cortan a dos secantes en segmentos proporcionales y dos de estas rectas son paralelas entonces las tres rectas son paralelas.
Demostraci
on. Sean AA0 , BB 0 y CC 0 rectas que cortan a dos secantes en los puntos A, A0 , B,
AB
A0 B 0
B 0 , C, C 0 respectivamente, tales que AA0 k CC 0 y
= 0 0 . Por el punto B tracemos una
BC
BC
recta paralela a AA0 , la cual interseca a A0 C 0 en el punto D (ver figura 71). Entonces, por el
A0 D
A0 B 0
A0 D
AB
=
. De donde, 0 0 =
, as por las propiedades
Teorema de Thales se tiene que:
BC
DC 0
BC
DC 0
0 0
0 0
AC
AC
, por lo que B 0 C 0 = B 0 D + DC 0 = DC 0 y por tanto B 0 D = 0,
de las proporciones 0 0 =
B0 C
DC 0
o equivalentemente B = D y por lo tanto, BB 0 k AA0 .

Figura 71: Recproco del Teorema de Thales


Corolario (Recproco del Teorema de Thales en el tri
angulo.) Si una recta intercepta
dos lados de un triangulo en segmentos proporcionales entonces la recta es paralela al tercer
lado del triangulo.
Tri
angulos semejantes. Decimos que el 4ABC es semejante al 4A0 B 0 C 0 (Ver figura 72), lo
cual denotamos as ABC A0 B 0 C 0 , si:
AB
AC
BC
=
=
A0 B 0
A0 C 0
B0C 0
y
BAC = B 0 A0 C 0 , ABC = A0 B 0 C 0 , ACB = A0 C 0 B 0 .
En los tres teoremas que se muestran a continuacion (los cuales son una consecuencia directa del
Teorema de Thales) se establecen las condiciones mnimas para demostrar que dos triangulos
son semejantes, a los cuales denominaremos: Criterios de Semejanza de Triangulos.
Primer criterio de semejanza de tri
angulos: Angulo-Angulo A-A. Si dos angulos de
un triangulo son congruentes con dos angulos de otro triangulo, entonces los dos triangulos son
semejantes.
47

Figura 72: Definicion de Semajanza de Triangulos.


Demostraci
on. Supongamos que en el 4ABC y 4A0 B 0 C 0 se tiene que ABC = A0 B 0 C 0
y ACB = A0 C 0 B 0 , entonces BAC = B 0 A0 C 0 (Por la suma de angulos internos en un
triangulo).
Sea D AB y E AC tales que AD = A0 B 0 y AE = A0 C 0 , dado que DAE = BAC =
B 0 A0 C 0 , se sigue por L-A-L que 4ADE = 4A0 B 0 C 0 , por consiguiente ADE = A0 B 0 C 0 =
ABC, de donde DE k BC (por ser iguales los angulos correspondientes) y por el teorema de
Thales
AC
AB
=
AD
AE
y por consiguiente
AC
AB
= 0 0
(1)
0
0
AB
AC
Sea F BC tal que DF k AC, entonces F C = DE = B 0 C 0 (porque DECF es paralelogramo
y por ser 4ADE = 4A0 B 0 C 0 ) y por el teorema de Thales
BA
BC
=
DA
FC
o lo que es lo mismo
AB
BC
= 0 0
0
0
AB
BC

(2)

Luego, de (1) y (2) se tiene que:


AB
AC
BC
=
=
.
A0 B 0
A0 C 0
B0C 0
As, se ha demostrado que los tres pares de angulos son congruentes y los tres pares de lados
son proporcionales, por lo tanto, 4ABC 4A0 B 0 C 0 .
Segundo criterio de semejanza de tri
angulos: L-A-L. Si un angulo de un triangulo es
congruente con otro angulo de otro triangulo y los lados que comprenden al angulo en el primer
triangulo son respectivamente proporcionales a los lados que comprende al angulo en el segundo
triangulo, entonces los dos triangulos son semejantes.
AB
AC
Demostraci
on. Suponga que el BAC = B 0 A0 C 0 y que 0 0 = 0 0 . Considere los punAB
AC
tos D y E, como en la demostracion del teorema anterior. Entonces por el criterio L-A-L,

48

4ADE = 4A0 B 0 C 0 , de lo cual se deduce que ADE = A0 B 0 C 0 . Por otra parte teneAB
AC
mos que:
=
, y al aplicar el recproco del teorema de Thales, se puede afirmar que
AD
AE
DE k BC, de lo cual a su vez se deduce que ADE = ABC, por angulos correspondientes
entre paralelas. Finalmente por transitividad se concluye que ABC = A0 B 0 C 0 . Por lo tanto,
4ABC 4A0 B 0 C 0 (Por el criterio A-A.)
Tercer criterio de semejanza de tri
angulos: L-L-L. Si los tres lados de un triangulo son
respectivamente proporcionales a los tres lados de otro triangulo, entonces los dos triangulos
son semejantes.
AC
BC
AB
Demostraci
on. Por hipotesis se tiene que: 0 0 = 0 0 = 0 0 y como antes sean D y E
AB
AC
BC
puntos sobre AB y AC respectivamente tales que AD = A0 B 0 y AE = A0 C 0 . Entonces por
el recproco del teorema de Thales se tiene que DE k BC y por consiguiente el ABC =
ADE y el ACB = AED, de donde 4ABC 4ADE (por el criterio A-A). Por ende
BC
BC
BC
AB
=
, luego por transitividad
= 0 0 , de donde DE = B 0 C 0 . En consecuencia
AD
DE
DE
BC
4ADE = 4A0 B 0 C 0 (por el criterio L-L-L), de lo cual se sigue que A0 B 0 C 0 = ADE y
A0 C 0 B 0 = AED, y por transitividad A0 B 0 C 0 = ABC y A0 C 0 B 0 = ACB =. Por lo
tanto, 4A0 B 0 C 0 4ABC (Por el criterio A-A.)

Ejercicios
1. Sean AB y CD las bases del trapecio ABCD, cuyas diagonales son perpendiculares. Si
se sabe que AD = 13, AE = 12 y CE = 4 encuentre las longitudes de CD y AB.
2. En la figura 73, el 4ABC es equilatero, sus lados tienen longitud 3 y P A es paralela a
BC. Si P Q = QR = RS, encontrar la longitud de CS.

Figura 73
3. Sea ABCD un trapecio de bases BC y AD, sus diagonales se cortan en E. Si BE = 3,
ED = 4 y CE = 2, determine la medida de AE.
4. Las bases de un trapecio miden 3 y 5, y si su altura mide 4. Encontrar la distancia desde
el punto de corte de las diagonales hasta la base mayor.

49

5. En la figura adjunta, el 4ABC es rectangulo en A y el 4ADB


es rectangulo en D. El punto E es el punto de interseccion de
los segmentos AD y BC. Si AC = 15, AD = 16 y BD = 12,
calcule el area del 4ABE.

6. El 4ABC es rectangulo en B. Se dibuja un rectangulo BEDF con D sobre la hipotenusa,


1
BC
= 1DE
.
E y F sobre BC y AB, respectivamente. Si AB = 1, demuestre que BE
7. Considerese los puntos A, B, C y D tales que A y B estan sobre el segmento OC y
OD respectivamente, donde O es el centro de la circunferencia de radio r (Ver figura
74). Si OA
OC = r2 = OB OD, demuestre que el 4AOB ' 4DOC y que CD =


r2
AB.17
OA OB

Figura 74
8. Sobre la circunferencia de centro O, se trazan los diametros AB y CD tales que AB CD.
Sea P un punto sobre el arco CBD y Q el punto de interseccion de las cuerdas AP y
CD. Si DO = 1, demuestre que AP AQ = 2.
9. Un segmento de recta AB es divido por los puntos interiores K y L de manera que
AL2 = AK AB. Sea P un punto exterior al segmento AB tal que AP = AL. Pruebe que
KP L = LP B. Figura 75.

Figura 75
17

La medida del segmento CD se denomina Distancia Inversa.

50

10. En la figura 76, AB y AC son tangentes a la circunferencia, y CE BD, siendo BD un


diametro. Probar que BE BO = AB CE.

Figura 76
11. Demostrar que

1
1
1
+
=
si se cumple que AX k BY k CZ. (Ver figura 77.)
AX BY
AZ

Figura 77
12. En la figura 78, el 4ABC es rectangulo. Se construyen exteriormente los cuadrados
ABEF y BCP Q. Demostrar que BM = BN .

Figura 78: .

51

13. Sean O, P y R los centros de las tres circunferencias. Si OR = r y Q es la interseccion


de P O con la circunferencia de centro R, demuestre que OP OQ = r2 . Ver figura 79.

Figura 79
14. Si en un triangulo rectangulo se traza la altura correspondiente a la hipotenusa, entonces:
a) Los dos nuevos triangulos que resultan, son semejantes entre si y semejantes al
triangulo original.
b) La altura es media proporcional
hipotenusa.

18

entre los segmentos que ella determina sobre la

c) Cada cateto es media proporcional entre la hipotenusa y la proyeccion del cateto


sobre la hipotenusa.
d ) Demuestre el teorema de Pitagoras.
15. Si dos triangulos tienen sus lados respectivamente paralelos o respectivamente perpendiculares, entonces los dos triangulos son semejantes.
16. Las alturas, las bisectrices y las medianas homologas de dos triangulos semejantes estan
en la misma razon que sus lados homologos.
BC
CA
17. Sean ABC y A0 B 0 C 0 dos triangulos semejantes con AAB
0 B 0 = B 0 C 0 = C 0 A0 = k. Demuestre
que: la razon entre los permetros de los triangulos es k y que la razon entre sus areas es
k2.

18. Teorema de Menelao. Dado el 4ABC, sea P un punto sobre la recta AB, Q un punto
sobre la recta BC, R un punto sobre la recta CA. Si los puntos P , Q, R estan alineados
AP BQ CR
entonces
= 1.
P B QC RA
Para demostrar este teorema, sea W un punto sobre la recta P QR tal que BW k AC:
a) Demuestre que los triangulos AP R y BP W son semejantes.
Si b es una magnitud tal que ab = cb , entonces decimos que b es media proporcional entre a y c,o de manera
equivalente: b es media proporcional entre a y c si y solo si b2 = a c.
18

52

Figura 80: Teorema de Menelao.


b) Demuestre que los triangulos CQR y BQW son semejantes.
AP BQ CR
c) De los literales a) y b) deduzca que
= 1.
P B QC RA

19. Teorema de Ceva. Dado el 4ABC, sea P un punto sobre el


recta AB, Q un punto sobre la recta BC y R un punto sobre
la recta CA. Si las rectas AQ, CP , BR concurren, entonces
AP BQ CR
= 1.
P B QC RA

Para demostrar este teorema, sean W y V los puntos de interseccion de la recta que pasa
por B paralela a AC, con las rectas CP y AQ, respectivamente.
a) Demuestre que 4AP C 4BP W y que 4AQC 4V QB.
b) Demuestre que 4BW P 4RCP y que 4BV P 4RAP .
AP BQ CR
= 1.
c) Utilice los literales a) y b) para probar que
P B QC RA
20. Si dos cuerdas se interceptan en el interior de una circunferencia entonces el producto
de las medidas de los segmentos determinados por el punto de interseccion en una de las
cuerdas es igual al producto de las medidas de los segmentos determinados en la otra
cuerda.
21. Si dos segmentos se interceptan en un punto que esta en el interior de los dos segmentos
y el producto de las medidas de los segmentos determinados por el punto de interseccion
en el primer segmento es igual al producto de las medidas de los segmentos determinados
por el punto en el segundo segmento,entonces los extremos de los segmentos estan sobre
una circunferencia.
22. Si desde un punto P exterior a una circunferencia se trazan dos semirrectas secantes
que cortan a la circunferencia en los puntos A, B y C, D respectivamente, entonces
P A P B = P C P D.

53

23. Si desde un punto P se trazan dos semirrectas con los puntos A, B sobre una y los puntos
C, D sobre la otra, tales que P A P B = P C P D, entonces los puntos A, B, C, D estan
sobre una circunferencia.
24. Si desde un punto exterior a una circunferencia se trazan dos semirrectas, una tangente
y la otra secante, entonces el segmento entre el punto y el punto de tangencia es media
proporcional entre los segmentos determinados entre el punto exterior y los puntos de
interseccion de la secante con la circunferencia. 19
25. Si P es un punto sobre el mismo plano que una circunferencia de centro O y radio r, y d
es la distancia del punto P al centro O de la circunferencia, demuestre que:
a) Si P esta en el interior de la circunferencia, entonces la potencia de P es r2 d2 .
b) Si P esta en el exterior de la circunferencia, entonces la potencia de P es d2 r2 .
c) Si P esta sobre de la circunferencia, entonces la potencia de P es cero.
26. (*) (IV OMCC, P-4) Sea ABC un triangulo, D el punto medio de BC, E un punto sobre
el segmento AC tal que BE = 2AD y F el punto de interseccion de AD con BE. Si
CAD = 60, encuentre la medida de los angulos del 4F EA.
27. (*) Sea ABCD es un trapecio con AD k BC. M y N son los puntos medios de CD y BC,
M
= 14 , demuestre que
respectivamente, y P el punto com
un de las rectas AM y DN . Si PAP
ABCD es paralelogramo.
28. Dado el 4ABC se construye un cuadrado P QRS con P en AB, Q en AC, R y S en BC.
Sea H el pie de la altura desde A hacia BC. Demuestre que:
1
1
1
=
+
PQ
AH BC
b) (ABC) = 2(P QRS) si y solo si AH = BC.

a)

29. Sea P un punto en el interior del 4ABC. Se trazan por P las paralelas a los lados del
triangulo, que queda dividido en tres triangulos y tres paralelogramos. Si las areas de los
tres triangulos de la subdivision son, en alg
un orden, 9, 16 y 25, hallar el area del 4ABC.
Problemas de Refuerzo.

30. (*) En la figura anexa, BC = CD = DE =


EA = x y AEB = 90. Demuestre que
ABC + ACD + ADE = 90.

19

Los problemas anteriores nos permite establecer la siguiente definicion de Potencia de un punto con
respecto a una circunferencia: La potencia de un punto P con respecto a una circunferencia de centro O y
radio r es el producto P A P B, donde A y B son los puntos de interseccion de la circunferencia con una recta
que pasa por P .

54

31. Las tres circunferencias de la figura 81 tienen el mismo radio r, sus centros son colineales
y la circunferencia de centro O2 es tangente a las otras dos. Por A se traza una tangente
a la circunferencia de centro O3 . Obtenga el valor del segmento BC en funcion de r.

Figura 81
32. Sea ABCD un rombo, con AC = 6 y BD = 8. Se construyen exteriormente los cuadrados
ADEF y CDHG, cuyos centros son O1 y O2 , respectivamente (Vea figura 82). Calcular
la medida del segmento O1 O2 .

Figura 82
33. Dado un paralelogramo ABCD, se trazan dos circunferencias tangentes externamente
en P , y tales que la primera es tangente internamente al ABC y la otra es tangente
internamente al CDA, como en la figura 83. Demuestre que B, P y D estan alineados.
34. Sea ABCD un cuadrado con P y Q sobre AB y BC tales que BP = BQ. Sea H el pie
de la perpendicular de B a P C. Demuestre que DHQ = 90.
55

Figura 83
35. En un 4ABC el CAB = 120. Encuentre la medida de la bisectriz interna del CAB
en funcion de los lados adyacentes.
36. El 4ABC tiene lados de 13, 14 y 15 unidades. El 4A0 B 0 C 0 esta dentro del 4ABC con
lados paralelos a los de este y a 2 unidades de distancia de los lados del mismo. Calcule
(ABC) (A0 B 0 C 0 ).
37. (*) (Asiatico Pacfica) Sea ABC un triangulo y D el pie de la altura con respecto a A.
Sean E y F puntos en una recta que pasa por D (distintos de D) tales que AE CE
y AF BF . Sean M y N los puntos medios de BC y EF , respectivamente. Demuestre
que AN N M .

56

7.

Puntos y Rectas Notables del Tri


angulo.

MEDIANAS
Definici
on: En un triangulo, una mediana es el segmento de recta que une un vertice con el
punto medio del lado opuesto.
Teorema: Las tres medianas de un triangulo concurren en un punto llamado el Centroide 20
del triangulo y usualmente es denotado por G. Ademas, las medianas de cortan mutuamente
en razon 2:1.
Demostraci
on: Dado el 4ABC sean A0 , B 0 , C 0 , los puntos medios de BC, CA, AB, respectivamente. Defina G como la interseccion de BB 0 con CC 0 . Por el teorema de la base media,
B 0 C 0 k BC y 2B 0 C 0 = BC; observe que 4BCG ' B 0 C 0 G, con razon de semejanza 2, por lo
que
GC
GB
=
=2
0
GB
GC 0
Analogamente, si G = AA0 BB 0 se cumple
G A
G B
=
=2
G B 0
G A0
As, G y G dividen al segmento BB 0 en dos segmentos cuya razon es 2:1, por lo que G = G ,
lo cual implica que AA0 , BB 0 , CC 0 concurren y
GB
GC
GA
=
=
=2
0
0
GA
GB
GC 0

MEDIATRICES
Definici
on: Dado un segmento AB, la mediatriz del segmento es el lugar geometrico de puntos
que equidistan de A y B, i.e., un punto P esta sobre la mediatriz de AB si y solo si P A = P B.
Teorema: La mediatriz de AB es una recta l perpendicular a AB y que pasa por su punto
medio.
Demostraci
on: Sea M el punto medio de AB, y l pasa por M y l AB. En primer lugar
se probara que todos los puntos de l satisfacen la definicion de mediatriz: Por definicion de
punto medio M A = M B. por lo que claramente M pertenece a la mediatriz de AB; sea P un
punto de l distinto de M , por criterio LAL, 4P M A 4P M B por lo que P A = P B. Ahora,
cabe preguntarse si existe alg
un punto fuera de l que tambien cumpla la definicion: suponga
P 0 tal que P 0 A = P 0 B, esto implica que 4P 0 AB es isosceles, y entonces P 0 AB = P 0 BA;
si M 0 es la proyeccion de P 0 sobre AB, por criterio ALA 4P 0 AM 0 4P 0 BM 0 , lo cual implica que M 0 A = M 0 B, es decir que M 0 = M , y esto obliga a que P 0 este sobre l (ya que P 0 M 0 = l).
Teorema: Las mediatrices de un 4ABC concurren en un punto que equidista de los vertices
del triangulo, llamado el Circuncentro del 4ABC
20

Tambien conocido como Geocentro, Centro de Gravedad, Baricentro, o mas formalmente Equibaricentro.

57

Figura 84: Concurrencia de Mediatrices, Circuncentro y Circuncrculo.


Usualmente, el circuncentro es denotado por O, y R representa la distancia del circuncentro a
los vertices
R = OA = OB = OC
A esta distancia se le llama Circunradio del 4ABC. As, O es el centro de una circunferencia
que pasa por A, B, C, cuyo radio es R, llamada el Circuncrculo del 4ABC.21
Demostraci
on: Sea O la interseccion de las mediatrices de AB y BC, por el teorema anterior,
como O pertenece a la mediatriz de AB se cumple OA = OB, y como tambien pertenece a la
mediatriz de BC, OB = OC; entonces OC = OA, y utilizando de nuevo el teorema anterior,
O debe pertenecer a la mediatriz de CA. As, las tres mediatrices concurren en O, y este punto
equidista de los vertices del 4ABC.
Corolario: Dado un triangulo, existe una circunferencia que pasa por los tres vertices (el circuncrculo); ademas, esta circunferencia es u
nica.
Una observacion importante es que la mediatriz del lado de un triangulo NO siempre pasa por
el vertice opuesto; de hecho, esto solo se da si el triangulo es isosceles.

ALTURAS
La altura es un concepto que esta intrnsecamente relacionado con la distancia de un punto a
una recta; la altura es la recta que debe trazarse para determinar esta distancia, i.e., es una
recta que pasa por el punto y es perpendicular a la recta. A la interseccion entre la altura y la
recta generalmente se le llama pie de la altura, o tambien (mas formal) proyeccion del punto
sobre la recta. En particular, para triangulos, definiremos la altura de la siguiente forma:
Definici
on: Dado un triangulo, una altura es una recta que pasa por un vertice y es perpendicular al lado opuesto.
21

En ocasiones, denotaremos a esta circunferencia por (ABC).

58

Es importante observar que el pie de la altura NO siempre pertenece a un lado; de hecho, una
altura puede estar al interior de un triangulo, coincidir con un lado, o estar completamente
afuera de un triangulo.
Teorema: Las alturas de un triangulo concurren en un punto, llamado el Ortocentro del triangulo, usualmente denotado por H.22
Demostraci
on: Dado el 4ABC, se construyen los puntos A1 , B1 , C1 , tales que ABA1 C,
BCB1 A, CAC1 B son paralelogramos. Observe que el 4ABC es el triangulo medial del 4A1 B1 C1 ,
y que las alturas del 4ABC son las mediatrices del 4A1 B1 C1 ; como las mediatrices de un
triangulo concurren (en este caso, las del 4A1 B1 C1 ), las alturas del 4ABC concurren.
La altura tambien puede escribirse en terminos de lugar geometrico:
Teorema: La recta l es perpendicular a AB si y solo si AL2 LB 2 es constante. Es decir, que
una recta perpendicular a AB es el lugar geometrico de los puntos L que satisfacen la condicion
anterior.
Demostraci
on: sea P la interseccion de l con AB, y L un punto arbitrario sobre l; por Pitagoras se tiene AL2 LB 2 = AP 2 P B 2 , y el termino derecho de la igualdad es constante. La
otra direccion de la implicacion se prueba por contradiccion.
De esa definicion tambien puede fabricarsele una demostracion del teorema anterior, sin embargo, no se aborda porque la prueba se basa en un resultado sofisticado llamadado Teorema
de Steiner.23

BISECTRICES
Definici
on: La bisectriz de un angulo es una recta que divide al angulo en dos angulos de
igual magnitud.
Teorema: El lugar geometrico de puntos que equidistan de dos rectas dadas, generan un par
de rectas perpendiculares llamadas bisectriz interna y bisectriz externa del angulo formado por
las rectas.
Demostraci
on: Suponga que las rectas se cortan en un punto O; sean a, b las rectas dadas, y
P un punto que equidista de ellas; si A y B son las proyecciones de P sobre a y b, respectivamente, entonces P A = P B. Observe que por criterio LLL (utilizando Pitagoras previamente),
4OAP 4OBP , por lo que P OA = P OB, i.e., P pertenece a la bisectriz del AOB.
Claramente aqu se dan dos casos, recuerde que para definir el angulo entre a y b se utilizan u
nicamente semi-rectas, por lo que las rectas a y b definen cuatro angulos, que por parejas pueden
22

El tri
angulo formado por los pies de las alturas de un 4ABC es llamado el tri
angulo
ortico del 4ABC.
Sean l, m, n, tres rectas perpendiculares a los lados del AB, BC, CA del 4ABC, respectivamente. Sean
L, M , N , puntos arbitrarios sobre l, m, n, respectivamente. Entonces las rectas l, m, n concurren si y s
olo si
AL2 + BM 2 + CN 2 = N A2 + LB 2 + M C 2 .
23

59

ser opuestos por el vertice o suplementarios; de estos se escoge cualquiera de ellos como referencia, entonces, si AOB coincide con este o con el opuesto por el vertice, la recta P O es llamada
bisectriz interna, y en caso contrario, bisectriz externa. As, el lugar geometrico son dos rectas,
y su perpendicularidad se basa en los pares de angulos que son suplementarios. Finalmente, si
a k b, el lugar geometrico es una recta paralela a a y b que se ubica entre ellas a igual distancia de
ambas (este es un caso extra
no de bisectriz interna, sin embargo, en ocasiones es u
til tener esta
convencion en mente; peor a
un, la bisectriz externa es una recta ideal llamada recta al infinito).
Teorema: Las bisectrices internas de un 4ABC concurren en un punto, llamado el Incentro
del 4ABC, usualmente denotado por I. La distancia de I a los tres lados del triangulo es igual
a un n
umero r, llamado el Inradio del 4ABC, y de aqu que la circunferencia de centro I y
radio r sea tangente a los lados del triangulo; dicha circunferencia es llamada el Incrculo del
4ABC.24

Figura 85: Concurrencia de Bisectrices Internas, Incentro e Incrculo.


Demostraci
on Sea I la interseccion de las bisectrices internas de A y B (obviamente, I
esta en el interior del 4ABC); como I pertenece a la bisectriz interna del A, por el teorema
anterior dist(I, AB) = dist(I, AC), y analogamente, como I pertenece a la bisectriz interna del
B, dist(I, AB) = dist(I, CB); entonces dist(I, AC) = dist(I, CB), y de nuevo por el teorema
anterior y dado que I esta al interior del triangulo, I pertenece a la bisectriz interna del C.
As, las tres bisectrices internas concurren en un punto que equidista de los lados del triangulo.
Es importante notar que las interseccion de una bisectrices interna con el lado opueto del
triangulo NO siempre coincide con el puntos de tangencia del incrculo;25 de hecho, esto ocurre
solamente si el triangulo es isosceles.
Corolario: Dado un triangulo, existe una circunferencia que es tangente interiormente a los
tres lados (el incrculo); ademas, esta circunferencia es u
nica.26
24

En algunas ocasiones denotaremos al incrculo por (ABC).


En la figura, el 4ABC es llamado tri
angulo tangencial del 4DEF .
26
Existen 3 circunferencias m
as que son tangentes a los tres lados del triangulo, llamados excrculos; estas
circunferencias se ubican en el exterior del triangulo.
25

60

Ejercicios
1. Las areas de los seis triangulos AGB 0 , AGC 0 , BGA0 , BGC 0 , CGA0 , CGB 0 son iguales e
iguales a un 61 del area del triangulo ABC.

Figura 86
2. Los cuatro triangulos AB 0 C 0 , BC 0 A0 , CA0 B 0 , A0 B 0 C 0 ,27 son congruentes entre si y semejantes al 4ABC con razon de semejanza 12 .
3. El centroide del 4ABC coincide con el centroide del triangulo medial 4A0 B 0 C 0 . Ademas,
estos dos triangulos tienen lados correspondientes paralelos (triangulos homoteticos).
4. En la figura 87, G es el centroide. Si GD = 2 y el area sombreada vale 5, calcule AD y el
(ABC).

Figura 87
5. Demostrar que las paralelas a los lados de un 4ABC, trazadas por el centroide G dividen
cada lado en tres partes iguales.
6. ABCD es un paralelogramo de centroide (baricentro) E, M es el punto medio de AD,
y F es la interseccion de AC con BM . Si el area de ABCD es 1, calcule el area del
cuadrilatero DEF M .
7. En el 4ABC, se traza la mediana AM . Demostrar que si BM = AM , entonces el
triangulo es rectangulo en A.
27

El 4A0 B 0 C 0 es llamado el tri


angulo medial del 4ABC.

61

8. La suma de las distancias del centroide a los puntos medios de los lados de un triangulo
es 20. Calcule la suma de las medianas del triangulo.
9. La mediana tiene longitud menor que la semisuma de los lados adyacentes, es decir AA0 <
b + c, BB 0 < c + a, CC 0 < a + b.
10. Dado el 4ABC, sean D y E puntos variables sobre los lados AB y AC respectivamente
tales que BC k DE. Entonces, la mediana AA0 puede definirse como el lugar geometrico
de los puntos P tales que P CD BE.28
11. Siempre es posible construir un triangulo XY Z con las medianas AA0 , BB 0 , CC 0 de un
4ABC dado. Ademas, los segmentos que unen el centroide del 4XY Z con sus vertices
son iguales a la mitad de los lados del 4ABC.
12. En el 4ABC, AB = BC y la mediatriz de BC interseca a la mediana BM en L. Si
LCB = 25, determine la medida del LAC.
13. Ley del Seno. Dado un 4ABC, se cumple que
senB
senC
1
senA
=
=
=
a
b
c
2R
14. Las reflexiones de H con respecto a los lados del 4ABC caen sobre el circuncrculo del
mismo, es decir HHa = Ha X y analogo para los otros lados.

Figura 88
15. Las reflexiones de H con respecto a los puntos medios de los lados del triangulo, caen
sobre el circuncrculo del mismo.
28

Si D = A se define P = A, y cuando D = B entonces P es punto medio de BC.

62

16. Si O y H son el circuncentro y el ortocentro de un 4ABC, respectivamente, entonces


BAH = CAO.
17. La altura AHa es bisectriz del Hb Ha Hc .
18. Los circuncrculos de 4ABC, 4ABH, 4BCH, 4CAH tienen igual radio.
19. La perpendicular trazada desde A al lado Hb Hc del triangulo ortico, pasa por el circuncentro del 4ABC.
20. A, B, C y H forman un cuadrilatero ortocentrico, es decir que cada punto es el ortocentro
del triangulo formado por los otros tres.
21. El ortocentro de un triangulo esta al interior, sobre un vertice, o afuera del triangulo, si
el triangulo es acutangulo, rectangulo, u obtusangulo, respectivamente.
22. El circuncentro del 4ABC es el ortocentro del triangulo medial 4A0 B 0 C 0 .
23. Sea O el circuncentro del 4ABC. Si AOC = 100 y OCB = 30, determine la medida
de los angulos del 4ABC.
24. Hallar los angulos de un triangulo cuyo triangulo ortico tiene angulos de 20, 50 y 110.
25. Sea ABC un triangulo obtusangulo de circuncentro O y altura AD. Si OAB = 25 y
OCB = 15, calcule el DAB.
26. El 4ABC de circuncentro O y altura BD. Si DAB = 35 y OBD = 10 encontrar los
angulos del triangulo ABC.
27. En la figura 89, AB es diametro de la circunferencia. Si X es la interseccion de CG con
AB, calcular el CXB.

Figura 89

63

28. En el 4ABC, se trazan la altura AH y la mediana BM . Demuestre que el 4M HC es


isosceles.
29. Un 4ABC es rectangulo en C, A = 75 y CH es altura. Demuestre que CH =

AB
.
4

30. Sea O el circuncentro del 4ABC con C = 45 y sea D el pie de la altura desde A.
Calcule la medida del ODC.
31. Dado el 4ABC isosceles con A = 90, sean P y Q son puntos dentro del triangulo tales
que BP = AQ y AP = CQ. Si BP y CQ se cortan en R, demostrar que AR P Q.
32. Se ubican los puntos M y K sobre los lados BC y CD del cuadrado ABCD, respectivamente, de modo que M C = KD. Sea P la interseccion de M D y BK, demuestre que
AP M K.
33. Sean D, E, F los puntos de tangencia del incrculo sobre los lados BC, CA, AB del
4ABC. Demuestre que se cumplen las siguientes relaciones, donde s denota el semipermetro del triangulo:
AE = AF = s a
BD = BF = s b
CD = CE = s c
34. El ortocentro del 4ABC es el incentro de su triangulo ortico.
35. Dado un 4ABC, su triangulo ortico y su triangulo tangencial tienen lados correspondientes paralelos (triangulos homoteticos).
36. Las bisectrices exteriores de B y C, junto con la bisectriz interior de A, concurren
en un punto, llamado el Excentro con respecto al vertice A, usualmente denotado por Ia .
Este punto es equidistante a los lados del 4ABC, dicha distancia es el Exradio respecto a
A, usualmente denotado por ra . As, la circunferencia de centro Ia y radio ra es tangente
exteriormente a los lados del 4ABC, y es llamada el Excrculo respecto a A.29
37. I es ortocentro del 4Ia Ib Ic . Ademas se cumple:
AX = AZ = s
BX = BY = s b
CY = CZ = s c
38. En un 4ABC, la bisectriz exterior del ABC y la bisectriz exterior del BCA se cortan
en D. La paralela a BC por D corta a AC en L y a AB en M . Si LC = 5 y M B = 7,
hallar LM .
39. El 4ABC es rectangulo en A. Si I es el incentro, calcular BIC.
40. En un 4ABC, el ABC CAB = 90. Sean D y E los pies de las bisectrices interior
y exterior del BCA respectivamente. Demuestre que CD = CE.
29

An
alogamente se definene los excrculos con respecto a los otros vertices.

64

Figura 90
41. En el 4ABC, AB < AC, AD es bisectriz, y E es un punto en AB tal que el EDB = 90.
El punto F sobre AC es tal que el BED = DEF . Demuestre que el BAD = F DC.
42. En el 4ABC se trazan las bisectrices interiores BD y CE tales que D es el punto sobre
AC, E es el punto sobre AB, 2BDE = 3B y CED = 2B. Calcular los angulos del
4ABC.
43. Dado el 4ABC con A = 90, sea D el pie de la perpendicular desde A. Sean ademas I
y J los incentros respectivos de 4ABD y 4ACD. Demostrar que la bisectriz del BAC
es perpendicular a IJ.
44. Un triangulo es isosceles si cumple alguna de las siguientes condiciones:
a) Dos medianas son iguales.
b) Dos alturas son iguales.
c) Dos bisectrices son iguales.30
45. Teorema de la Bisectriz: Dado el 4ABC, sean P y P 0 sobre BC. Se cumple que AP
y AP 0 son la bisectriz interna y la bisectriz externa del A si y solo si
BA
BP 0
BP
=
= 0
PC
AC
PC
Sugerencia: Para demostrar la primera igualdad, trace CD k AP con E sobre la prolongacion de AB.
30

Este caso es aparentemente tan sencillo como los anteriores, pero realmente es un resultado muy complicado
y recibe el nombre de Teorema de Steiner-Lehmus.

65

46. (*) De acuerdo con los datos de la grafica 91, calcular el valor de AB.

Figura 91
47. Dos circunferencias son tangentes internamente en P , y una cuerda AB de la circunferencia de radio mayor es tangente en Q a la otra circunferencia. Ver figura 92.
a) Demuestre que P Q es bisectriz del AP B.
b) Llame A0 y B 0 a las otras intersecciones de P A y P B con la circunferencia de radio
menor y suponga que AB = 15, P A0 = 3 y P B 0 = 2; calcule AQ y BQ.

Figura 92
48. (*) Sea ABC un triangulo tal que las medianas respectivas a B y C son perpendiculares.
Demuestre que se cumple la relacion.
5BC 2 = CA2 + AB 2 .
49. (*) Teorema de Poncelet: Demuestre si 4ABC es un triangulo rectangulo con A =
90, entonces 2(r + R) = b + c.

66

Problemas de Refuerzo.
50. (*) Sea ABCD un paralelogramo. Q es el punto medio de AD, F el pie de la perpendicular
por B sobre QC. Probar que AF = AB.
51. Dado el rombo ABCD, se trazan las bisectrices internas de DAC, CAB, BCA,
ACD, y cortan a DC, CB, BA, AD en P , Q, R, S, respectivamente. Demuestre que
P QRS es un rectangulo.
52. (*) Sea ABCD un cuadrilatero tal que AB = CD. Las mediatrices de AC y BD se cortan
en P . Probar que P AC = P CA = P BD = P DB.
53. (*) ABC es un triangulo y P un punto en su interior. Sean A0 , B 0 y C 0 las reflexiones de
P sobre BC, CA y AB, respectivamente. D, E y F son los pies de las perpendiculares
respectivos desde A, B y C hacia B 0 C 0 , C 0 A0 y A0 B 0 . Probar que AD, BE y CF son
concurrentes.
54. (*) (Arnoldo Aguilar) En la figura 93, ABGH, BCF G y CDEF son cuadrados. Si I es
el centro de ABGH y J = DH BG, demuestre que I, J y F estan alineados.

Figura 93
55. (*) (Arnoldo Aguilar) Sea ABC un triangulo equilatero. M y N son los puntos medios de
AB y BC, respectivamente. Exteriormente al 4ABC se construye un triangulo rectangulo
isosceles 4AP C, con AP C = 90 . Si I es la interseccion de AN y M P , demuestre que
CI es la bisectriz de ACM .

56. (*) En la figura adjunta, el 4ABC es tal que A = 90 y


B = 60. Cual es el radio de la circunferencia?

57. (*) Dado el paralelogramo ABCD, sea M el punto medio de AB, y N la interseccion de
CD con la bisectriz interna del ABC. Demuestre que M C BN si y solo si AN es
bisectriz del DAB.
67

58. (*) En el 4ABC, se sabe que los vertices B, C, el circuncentro O y el ortocentro H del
4ABC estan todos sobre una misma circunferencia.
a) Calcule el valor de A.
b) Demuestre que el incentro tambien pertenece al circuncrculo de BCOH.
59. (*) Sea ABC un triangulo de ortocentro H. Sean P y Q los pies de las perpendiculares
desde H a las bisectrices interior y exterior de A, respectivamente. Si M es el punto medio
de BC, mostrar que P , Q y M estan alineados.
60. (*) En un triangulo ABC, sea M el punto medio de BC. Si se cumple que AB 6= AC y
ademas M AC + ABC = 90 , hallar BAC.
61. (*) Sea ABC un triangulo y U un punto de su circuncrculo tal que AU es bisectriz. Las
mediatrices en AB y AC cortan a AU en X y Y . Sea T la interseccion de BX con CY .
Demostrar que AU = T B + T C.
62. (*) (The 59th Romanian Mathematical Olympiad Final Round) Sea ABCD un rectangulo
de centro O con AB 6= BC. La perpendicular en O a BD corta a las lneas AB y BC en
los puntos E y F , respectivamente. Sean M y N los puntos medios de los segmentos CD
y DA, respectivamente. Probar que las lneas rectas F M EN .
63. (*) Sea ABC un triangulo rectangulo, con A = 90 . Sea D un punto en su interior tal
que DAC = DCA = DBC = , y AC = BD. Determine el valor de .
64. (*) Sea ABC un triangulo y M un punto tal que M AB = 10, M BA = 20, M AC =
40 y M CA = 30. Probar que el 4ABC es isosceles.
65. (*) En la figura 94, ABCD y P QRS son cuadrados, 4ABP 4BCQ 4CDR
4DAS y los los radios de las cinco circunferencias son iguales a r. Si a es el lado del
cuadrado ABCD, determine r en funcion de a.

Figura 94

68

66. (*) Recta de Euler. El centroide G, el ortocentro H y el circuncentro O de un triangulo


estan alineados, y ademas GH = 2GO.
67. Circunferencia de los 9 puntos:31 Dado un 4ABC de ortocentro H, se cumple que
los puntos medios de los lados, los pies de las alturas, y los puntos medios de HA, HB,
HC, se ubican sobre una misma circunferencia. Ademas, el centro de esta circunferencia
es el punto medio de HO y su radio es R2 , donde O y R son el circuncentro y el circunradio
del triangulo.
Para demostrar este resultado se sugiere seguir los siguientes pasos:
a) Si Ha es el pie de la altura trazada desde A, demuestre que la reflexion de H con
respecto a Ha pertenece a circuncrculo del 4ABC. Resultados similares se cumplen
para Hb y Hc .
b) Si A0 es el punto medio de BC, demuestre que la reflexion de H con respecto a A0
pertenece al circuncrculo del 4ABC.
c) De los resultados anteriores, observe que hay 9 puntos sobre el circuncrculo del
4ABC: los vertices, las reflexiones de H con respecto a los pies de las alturas, y
las reflexiones de H con respecto a los puntos medios de los lados; a partir de esto,
concluya que los puntos medios de los segmentos que van de H a estos 9 puntos,
tambien deben pertenecer en una misma circunferencia.
d) Concluya ademas que el centro de esta nueva circunferencia es el punto medio de HO.
Otro camino de solucion es el siguiente:
a) Sea 4A0 B 0 C 0 el triangulo medial del 4ABC. Pruebe que A0 B 0 C 0 = BHa C 0 y
concluya que Ha A0 B 0 C 0 es un cuadrilatero cclico; los mismo debe cumplirse para Hb
y Hc .
b) Sea X el punto medio de HA. Demuestre que B 0 A0 C 0 +B 0 XC 0 = 180 y concluya que
XC 0 B 0 A0 es un cuadrilatero cclico; lo mismo debe cumplirse para los puntos medios
de HB y HC.
c) De lo anterior, concluya que los pies de las alturas y los puntos medios de los segmentos
que van desde H hasta los vertices del 4ABC, se ubican sobre el circuncrculo del
4A0 B 0 C 0 .
d) Si N es el circuncentro del 4A0 B 0 C 0 , demuestre que N , O, G forman la recta de Euler
del 4A0 B 0 C 0 y utilice sus propiedades para probar que N es el punto medio de HO.
68. El area del 4ABC, denotada por [ABC], cumple:
base altura
2
ab senC
bc senA
ca senB
abc
[ABC] =
=
=
=
2
2
2
4R
[ABC] = sr
p
[ABC] =
s(s a)(s b)(s c). (F
ormula de Her
on).
[ABC] =

31

Tambien conocida como Circunferencia de Feuerbach.

69

69. El circunradio, el inradio y los exradios de un triangulo cumplen:


4R = ra + rb + rc r
[ABC] = ra (s a) = rb (s b) = rc (s c)
r
(s a)(s b)(s c)
r =
s
r
s(s b)(s c)
ra =
sa
r
s(s a)(s c)
rb =
sb
s
s(s a)(s b)
rc =
(s c)
70. Dado el 4ABC, sea I el incentro e Ia el excentro respecto a A.
a) Demuestre que BICIa es un cuadrilatero cclico.
b) Si M es la interseccion de IIa con el circuncrculo del triangulo (M 6= A), demuestre
que dicho punto es el circuncentro de BICIa .
c) Sea M 0 el punto diametralmente opuesto a M en el circuncrculo, y sea P la proyeccion
de I sobre AB. Demuestre que 4M 0 CM ' 4AIP .
d) Sea O el circuncentro del triangulo Calcule la potencia de punto de I con respecto
al circuncrculo, y utilizando los resultados anteriores deduzca la Formula de Euler :
OI 2 = R2 2Rr.
e) A partir de la formula de Euler demuestre que R 2r.

70

8.

Soluci
on a Problemas Selectos.

Teoremas Fundamentales del Tri


angulo.
1. En la figura 95, ABDE es un cuadrado y BCD es un triangulo isosceles con BD = DC.
Si ABC = 160, determinar la medida de AEC.

Figura 95
Soluci
on: DBC = DCB = 160 ABD = 70, de donde se obtiene que BDC = 40
y EDC = EDB + BDC = 130. Como 4EDC es isosceles, entonces DEC =
DCE = 25. Por lo tanto AEC = 90 DEC = 65.
2. Hallar la suma de los angulos +  + + en la figura 96.

Figura 96
Soluci
on: CAB = , EDC = por ser opuestos por el vertice. Como el AF D
externo en el 4BDF , se tiene AF D = + . Sumando los angulos internos del 4AEF
se tiene + + +  = 180.
3. (XV Competencia de Clubes Cabri Primera Ronda) En la figura 97, ABCD es un
rectangulo tal que AB = 2BC. M es el punto medio de AB y los triangulos AM E
y M BF son equilateros. Si P es la interseccion de las rectas DE y CF , encuentre los
angulos del 4CDP .
71

Figura 97
Soluci
on: Note que AD = AE = F B = BC por lo que 4DAE y 4BCF son ambos
isosceles. Luego DAE = CBF = 90+60 = 150 lo que implica que P DC = P CD =
90 15 = 75 y luego CP D = 30.
4. Sea ABC un triangulo rectangulo con CAB = 90 (Ver figura 98). D es un punto sobre
la prolongacion de BC tal que BD = BA. E es un punto en el mismo semiplano que A
respecto de BC, tal que CE BC y ademas CE = CA. Mostrar que A, D y E estan
alineados.

Figura 98
Soluci
on: Sea CBA = 2; el 4ABD es isosceles y BAD + BDA = 2, por lo
que BAD = BDA = . Como CAB = 90 entonces ACB + ABC = 90 y como
CE es perpendicular a BC entonces ECA + ACB = 90; por lo tanto, ABC =
ECA = 2. Con esto, como 4ECA es isosceles, CEA = CAE = 90 . Luego,
EAC + CAB + BAD = 180 y as E, A y D estan alineados.
5. Dado un cuadrado ABCD, se construyen los triangulos equilateros ABP (exteriormente)
y ADQ (interiormente). Probar que C, P y Q estan alineados. Figura 99.
72

Soluci
on: Observe que P AQ = BAD = 90 y P A = BA = DA = DQ, por lo
que 4P AQ es triangulo rectangulo isosceles, y por tanto, P QA = 45. Por otra parte,
QDC = 90 ADQ = 30 y QD = AD = CD, es decir, el 4CDQ es isosceles
con el angulo comprendido entre lados iguales de 30, por lo que DQC = 75. As,
P QA + AQD + DQC = 180 y por lo tanto, C, P y Q estan alineados.

Figura 99
6. En la figura 100, AB = BC = CD = DE = EF = F G = GA. Calcule la medida del
DAE. Referenciasfig20

Figura 100
Soluci
on: Sea DAE = . Como los triangulos ABC y AGF son isosceles, ACB =
AF G = . Calculando los angulos externos de 4ABC y 4AF G se tiene F BC =
CGF = 2. Como 4GF E y 4BCD son isosceles, GEF = BDC = 2. Calculando
angulos externos de 4ADC y 4AEF se obtiene ECD = DF E = 3. Como 4CDE
y 4F ED son isosceles, CED = F DE = 3. Entonces, la suma de los angulos internos
del 4AED da + 3 + 3 = 180, de donde = 180
.
7
7. (Etapa semifinal Estatal de XXII Olimpiada Mexicana de Matematicas) En la figura
101 se muestra un hexagono regular ABCDEF de lado 1. Los arcos del crculo que estan
dibujados tienen centro en cada vertice del hexagono y radio igual a la distancia al vertice
opuesto. P , Q, R, S, T y U son los puntos de corte de estos arcos. Cuanto mide cada
lado del hexagono P QRST U ?

73

Figura 101
Soluci
on: El hexagono P QRST U es regular y con el mismo centro que ABCDEF . Sea
O el centro de ambos (Vease Figura 65). El lado buscado es igual a OP . Tenemos que
CF = F P = P C = 2 por ser radios de los arcos dibujados; entonces CF P esequilatero
de lado 2 y OP es una altura de este triangulo que, por Pitagoras, es igual a 3.
8. (XXVIII Olimpiada Brasile
na de Matematica) En la figura 102, AB = AC, AM = AN
y CAM = 30, encuentre el valor del BM N .

Figura 102
Soluci
on: Como 4ABC y 4AM N son isosceles, sean ABC = ACB = y AM N =
AN M . Por la formula del angulo externo se tiene
ACM + M AC = AM B = AM N + BM N
+ 30
=
AN M + BM N
=
(N BM + BM N ) + BM N
=
+ 2BM N
Esto implica que BM N = 15.

74

Congruencia de Tri
angulos
1. En la figura 103, ABC es un triangulo equilatero y CDEF es un cuadrado. Se construye
un punto G tal que CF = CG y ademas CF G = 15. Probar que AGC = BDC.

Figura 103
Soluci
on: BCD = 180 ACB DCF = 30. Como 4GCF es isosceles, CGF =
CF G = 15 y ACG = CGF + CF G = 30. Por criterio LAL, 4BCD 4ACG,
por lo tanto BDC = AGC.
2. (Cuaderno de Olimpiadas Mexicanas - Geometra) En la figura 104, ABCD un cuadrado
y EF GH. Demuestre que que EF = GH.

Figura 104
Soluci
on: Se construyen EK y GM con K sobre CD y M sobre AD tales que EK k AD
y GM k CD. Luego se demuestra que 4EF K 4GHM , con EF = GH.
3. Sea ABCD un cuadrado. Se construyen triangulos equilateros ADP y ABQ como se
muestra en la figura 105. Sea M la interseccion de CQ con AD y N la interseccion de
CP con AB. Demuestre que CM N es un triangulo equilatero.
Soluci
on: Note que P D = AD, porque 4AP D es equilatero, y AD = CD porque
ABCD es cuadrado, por lo que P D = CD, es decir, el 4CDP es isosceles, con CDP =
CDA P DA = 30, entonces DP C = DCP = 75, y BCN = BCD DCP =
15. Analogamente, 4BCQ es isosceles con angulos 30, 75, 75, por lo que M CN =
75

Figura 105
BCQ BCN = 60. Finalmente, como la figura es simetrica con respecto a AC,
CM = CN , entonces, el triangulo CM N es equilatero porque tiene dos lados iguales y
un angulo interno igual a 60.
4. (Examen final de XVI Olimpiada mexicana de Matematica) Los angulos de un triangulo
ABC estan en progresion aritmetica (B A = C B = ), D, E, y F son los
puntos medios de los lados BC, CA y AB, respectivamente. Llamamos H al pie de la
altura trazada desde C (que cae entre B y F ) y G a la interseccion entre DH y EF .
Cuanto vale F GH?
Soluci
on: Note que A + B + C = 3A + 3 = 180, lo cual implica que A + =
60 = B. Entonces 4BCH es un triangulo 30, 60, 90, y dado que D es punto medio de
BC, el 4BDH es equilatero. Luego, como BC k EG, F GH = BDH = 60. Ver figura
106.

Figura 106
5. 4ABC es un triangulo isosceles con ABC = ACB = 80. D es un punto en AC tal
que ABD = 10. Demuestre que AD = BC. Figura 107.
Soluci
on: Se traza un punto D0 sobre AC tal que AD0 = BC. Se construye exteriormente
76

el triangulo equilatero AEB. Luego, AE = AB, D0 A = CB y EAD0 = ABC lo


cual implica que 4EAD0 4ABC, de donde se deduce que el 4D0 EB es isosceles
y BED0 = BEA D0 EA = 40. Se sigue que EBD0 = 70 y como D0 BA =
EBD0 ABE = 10, resulta que D0 = D y por lo tanto BC = AD0 = AD.

Figura 107

77

Cuadril
ateros.
1. Sea ABCD un paralelogramo. Se construyen triangulos equilateros exteriores 4CDP y
4ADQ, como se muestra en la figura 108. Demuestre que el 4BP Q es equilatero.

Figura 108
Soluci
on: Observe que al hacer una rotacion de centro P y angulo 60, el triangulo
P BC se transforma en el triangulo P QD (observe los segmentos P C y CB tras esta
transformacion), mientras que al hacer una rotacion de centro Q y angulo 60, el triangulo
P QD se transforma en triangulo BQA. Como la rotacion mantiene las distancias, P B =
P Q = BQ, por lo que el trangulo BP Q es equilatero.32
2. (II Olimpiada Matematica del Cono Sur) En la figura 109 ABCD y AECF son paralelogramos. Demuestre que BEDF es paralelogramo.

Figura 109
32

Una demostraci
on m
as rigurosa se basa en el calculo de los angulos P CB = P DQ = BAQ = 120 +
ABC y en la utilizaci
on del criterio LAL para justificar 4P CB 4P DQ 4BAQ.

78

Soluci
on 1: Sea M el punto medio de AC. Las diagonales AC y BD se bisecan en M ,
mientras que las diagonales AC y EF tambien se bisecan en M , entonces BD y EF se
bisecan en M por lo que BEDF es un paralelogramo.
Soluci
on 2: Como AD k CB y AE k CF entonces DAE = BCF . Entonces, por
propiedades de paralelogramos BAE = BAD EAD = BCD BCF = F CD;
ademas, AB = CD y AE = CF . Por criterio LAL, 4BAE 4DCF , y entonces BE =
DF . Analogamente se demuestre que 4ABF 4CDE, lo cual implica BF = DE.
Como BEDF es un cuadrilatero con lados opuestos iguales, es un paralelogramo.
3. ABCD es un cuadrilatero convexo y O es un punto en su interior. Sean P , Q, R, S,
los puntos medios de los lados AB, BC, CD, DA, respectivamente. Por P se traza una
paralela a OR, por Q se traza una paralela a OS, por R se traza una paralela a OP , y
por S se traza una paralela a OQ. Demuestre que estas cuatro rectas concurren.
Soluci
on: Al tomar las rectas OP y OR y sus paralelas se forma el paralelogramo P ORM ,
y al tomar las rectas OQ y OS y sus paralelas se forma el paralelogramo OQN S. Por el
teorema de Varignon, sabemos que P QRS es un paralelogramo, y llamaremos T al punto
de corte de sus diagonales. Observe que el punto de corte de las diagonales de P ORM es
el punto medio de P R, i.e., T ; analogamente, el punto de corte de las digonales de OQN S
es el punto medio de SQ, i.e., T nuevamente. As, M es la reflexion de O con respecto a
T , y de igual forma queda definido N , por lo que M = N y las cuatro rectas concurren.
4. (Hector Alberti) Sea ABCD un cuadrado. Se construyen los triangulos equilateros BDA0 ,
ACB 0 , BDC 0 y ACD0 (Vease figura 110). Demuestre que el A0 B 0 C 0 D0 es tambien un
cuadrado.

Figura 110
Soluci
on: Como A0 B = A0 D, AB = AD, CB = CD, C 0 B = C 0 D, los puntos A0 , A,
C, C 0 pertenecen a la mediatriz de BD, y por tanto, estan alineados. Analogamente, B 0 ,
B, D, D0 estan alineados; por lo tanto A0 C 0 B 0 D0 . Por otra parte, si O es el centro
de ABCD, como los triangulos equilateros construidos son todos iguales (tienen lados
iguales a la diagonal de ABCD) de altura h, OA0 = OB 0 = OC 0 = OD0 = h. Entonces,
A0 B 0 C 0 D0 es un cuadrilatero con diagones que se bisecan en O (es paralelogramo), son
iguales A0 C 0 = B 0 D0 = 2h (es rectangulo) y son perpendiculares (es rombo), lo cual
implica que es cuadrado.
79

5. Un trapecio isosceles tiene diagonales perpendiculares y su area es 2010, determine su


altura.
Soluci
on: Considere la figura 111. Sea ABCD el trapecio del problema (AB = CD),
como es trapecio isosceles, es simetrico con respecto a la mediatriz de las bases, en particular, AC = BD. Sean P y Q los pies de las perpendiculares a AD trazadas desde B y
C, respectivamente. Por LAL, 4ABD 4DCA lo cual implica CAD = BDA = 45
(debido a que AC BD) luego en el triangulo rectangulo 4ACQ, ACQ = 45 por lo
que AQ = CQ y es facil ver que BC = P Q. Luego
(CQ)(BC + AD)
2
(CQ)(P Q + AP + P Q + QD)
=
2
= (CQ)(AP + P Q)
= CQ2

2010 = (ABCD) =

Luego CQ =

2010.

Figura 111
6. (IX Competencia de Clubes Cabri, Segunda Ronda) Sea ABCDEF un hexagono regular
cuyo centro es O. Se construyen los cuadrados F SOP y ORCQ. Demuestre que AP QB
y SEDR son rectangulos. Figura 112.
Soluci
on: Por construccion P F = P O = SF = SO, y por propiedades de hexagono
regular33 AF = AO = EF = EO, entonces P , S A, E, pertenecen a la mediatriz de F O
y por tanto, estan alineados sobre una recta perpendicular a F O. Analogamente, Q, R,
B, D estan alineados sobre una recta perpendicular a CO; ademas, es facil demostrar
que AP = BQ = DR = ES. Observe ademas que AB k CF k DE, lo cual implica
(AB k CF ) (AP k BQ), es decir, AP QB es rectangulo, y analogamente para SEDR.

33

Los tri
angulos OAB, OBC, OCD, ODE, OEF , OF A son equilateros.

80

Figura 112
7. Sobre los lados del 4ABC se trazan exteriormente los cuadrados ABP Q, CARS y
BCT U . Luego se trazan los paralelogramos AQA0 R, CSC 0 T y BU B 0 P , como en la figura
113.
a) Sean A00 , B 00 , C 00 los centros de los cuadrados BCT U , CARS, ABP Q, respectivamente. Demuestre que estos centros estan sobre los lados del 4A0 B 0 C 0 .
b) Demuestre que AA00 , BB 00 , CC 00 concurren.
Soluci
on:
a) Observe que al hacer una rotacion de centro A00 y angulo igual a 90, el 4A00 U B 0 se
transforma en el 4A00 BA, y a la vez este u
ltimo se transforma en el 4A00 CC 0 (esto es
porque A00 U 7 A00 B 7 A00 C y U B 0 7 BA 7 CC 0 ); esto significa que A00 B 0 A00 A
y A00 A A00 C 0 , por lo que B 0 , A00 , C 0 estan alineados, es decir, A00 pertenece a B 0 C 0 .
Analogamente se prueban los otros casos.
b) De lo anterior, observe que AA00 es mediatriz de B 0 C 0 , por lo que AA00 , BB 00 , CC 00
concurren en el circuncentro del 4A0 B 0 C 0 .
8. Se dibujan cuadrados exteriores a los lados de un paralelogramo (Vea figura 114), demuestre que:
a) El cuadrilatero determinado por los centros de esos cuadrados es un cuadrado.
b) Las diagonales de ese cuadrado son concurrentes con las del paralelogramo.
Soluci
on:

81

Figura 113
a) Observe que al hacer una rotacion de centro O2 y angulo igual a 90, el 4O2 BO1 se
transforma en el 4O2 CO3 (observe que los segmentos O2 B y BO1 se transforman
en O2 C y CO3 , respectivamente), por lo que O2 O1 = O2 O3 y O2 O1 O2 O3 . Repitiendo este razonamiento, O1 O2 = O2 O3 = O3 O4 = O4 O1 y estos segmentos son
perpendiculares si son consecutivos, por lo que O1 O2 O3 O4 es un cuadrado.
b) Basta demostrar que AC y O1 O3 se bisecan,34 y esto es equivalente a demostrar
que AO1 CO3 es un paralelogramo. Esto es cierto porque AO1 = CO3 y AO1 k CO3
(ambos segmentos son perpendiculares a O1 B)

Figura 114
34

Porque as los puntos de corte de las diagonales de ABCD y O1 O2 O3 O4 coincidiran.

82

9. Dado un 4ABC, se construyen exteriormente los triangulos rectangulo isosceles 4ACP


y 4BCQ, con AC y BC como hipotenusas. Si M es el punto medio de AB, demuestre
que el 4M P Q tambien es un triangulo rectangulo isosceles.
Soluci
on: Construya los cuadrados exteriores ACDE y BCF G, como muestra la figura
115. Observe que P y Q son los puntos medios de AD y BF , respectivamente. Al rotar
el 4BCD con centro C y angulo de 90, se genera el 4F CA, entonces dichos triangulos
son congruentes y en por tanto BD = AF y BD AF . Por otra parte, observe que M P
es base media del 4BAD, por lo que 2M P = BD y M P k BD; analogamente, M Q es
base media del 4ABF , por lo que 2M Q = AF y M Q k AF . Por lo tanto M P = M Q y
M P M Q.

Figura 115

83


Angulos
en Circunferencia.
1. Dada la figura 116, demuestre que AB k A0 B 0 .

Figura 116
Soluci
on: Observe que los cuadrilateros ABQP y A0 B 0 QP son cclicos, por lo que
P AB = P QB 0 = 180P A0 B 0 , por lo tanto AB k A0 B 0 .
2. Dos circunferencias de centros O1 y O2 son tangentes (interna o externamente) en un
punto P ; por este punto se traza una recta que corta nuevamente a la circunferencias en
A y B, respectivamente. Demuestre que AO1 k BO2 .
Soluci
on: En la figura 117 se ha considerado que las circunferencias son tangentes exteriormente, sin embargo, el otro caso es analogo. Se sabe que O1 , P , O2 estan alineados, y
que 4AP O1 y BP O2 son triangulos isosceles (dos de sus lados son radios de una circunferencia), entonces O1 AP = AP O1 = BP O2 = O2 BP , por lo que AO1 k BO2 .

Figura 117
3. Dadas dos circunferencias una fuera de la otra como en la figura 118, demuestre que las
tangentes comunes externas forman segmentos iguales; analogamente, las tangentes comunes internas forman segmentos iguales.
Soluci
on:35 Sea P la interseccion de las tangentes comunes externas AA0 y BB 0 . Entonces
35

Suponemos que las circunferencias tienen radios distintos; cuando los radios son iguales, el problema se
justifica por la simetra de la figura.

84

AA0 = P A0 P A = P B 0 P B = BB 0 . Analogamente se resuelve el caso de las tangentes


comunes internas.

Figura 118
4. Teorema de Pithot. Demuestre que en todo cuadrilatero inscribible, la suma de lados
opuestos es igual.36
Soluci
on: Considere la figura 119, ABCD es el cuadrilatero inscribible, con P , Q, R, S,
los puntos de tangencia sobre AB, BC, CD, DA, respectivamente. Entonces
AB + CD = AP + P B + CR + RD
= AS + BQ + CQ + DS
= BC + DA

Figura 119: Teorema de Pithot.


5. Teorema de Steiner. En todo cuadrilatero exinscrito a una circunferencia, la diferencia
de las longitudes de lados opuestos es igual.
Soluci
on: El cuadrilatero puede quedar en posiciones como las de la figura ??; en ambos
casos, la demostracion es muy similar, y analoga a la de Pithot. Para la figura de la
36

El recproco de este teorema y del siguiente son tambien es ciertos.

85

izquierda se tiene que


AB CD = (AP BP ) (CR RD)
= (AS BQ) (CQ DS)
= AD BC

Figura 120: Teorema de Steiner.


6. Teorema de Miquel: Dado un 4ABC, sean X, Y , Z puntos sobre AB, BC, CA,
respectivamente . Demuestre que los circuncrculos de 4AXZ, 4BY X, 4CZY tienen
un punto en com
un M .
Soluci
on: Sea M el otro punto de corte de los circuncrculos de 4AXZ y 4BY X. Como
los cuadrilateros AXM Z y BY M X son cclicos, se tiene
Y M Z = 360 XM Z Y M X
= 360 (180 A) (180 B)
= 180 C
Entonces, CY M Z es cuadrilatero cclico, por lo que M esta sobre el circuncrculo del
4CZY .

Figura 121: Teorema de Miquel.

86

7. Sea ABC un triangulo, y sean L y N las intersecciones de la bisectriz del angulo A con
el lado BC y el circuncrculo de ABC respectivamente (Ver figura 122). Construimos la
interseccion M del circuncrculo de ABL con el segmento AC. Prueba que los triangulos
BM N y BM C tienen la misma area.
Soluci
on: Observe que ABN C y ABLM son cuadrilateros cclicos, por lo que N CB =
N AB = LAM = LBM , por lo que CN k BM . Entonces, las distancias de N y C a
la recta BM son iguales, y por tanto, el area del 4BM N es igual al area del 4BM C.

Figura 122
8. Sea AB el diametro de una semicircunferencia. Se colocan los puntos M y K sobre la
semicircunferencia y sobre AB, respectivamente.37 Sea P el centro de la circunferencia
que pasa por A, K y M ; sea Q el centro de la circunferencia que pasa por B, K y M .
Demuestre que M P KQ es concclico.
Soluci
on: Como AB es diametro, AM B = 90, entonces M AB + M BA = 90. As
M P K + M QK

= 2M AK + 2M BK
2 = 2 (M AB + M BA)
= 180

Por lo tanto, M P KQ es concclico.

37

M y K son distintos de A y B.

87

9. Las circunferencias 1 y 2 se cortan en los puntos A y B. Por el punto A se traza una recta
que corta nuevamente a las circunferencias 1 y 2 en los puntos C y D, respectivamente.
Por los puntos C y D se trazan tangentes a las circunferencias, las cuales se cortan en el
punto M . Demuestra que M CBD es cclico. Figura 123.
Soluci
on: Es suficiente probar que M CBD es un cuadrilatero con un par de angulos
opuestos suplementarios. Por angulos seminscritos y suma de angulos internos de un
triangulo, se tiene
CM D + CBD = CM D + CBA + DBA
= CM D + M CA + M DA
= 180

Figura 123
10. El 4ABC cumple que A = 90 y AB = AC. Se toma un punto E del segmento AB, se
construye interiormente un triangulo equilatero AEF . EF corta BC en I, y se construye
exteriormente un triangulo equilatero BIJ. Encuentre EJB.

Soluci
on: Como el BJI = 60 = AEI,
el cuadrilatero BEIJ es cclico, por lo que el
EJB = EIB = AEI EBI = 15.

88

11. En la figura 124, se sabe que AO1 B AO2 B = 70 y ademas la tangente EB forma
el triangulo isosceles ABE, con AB = AE. Encuentre EBC.

Figura 124
Soluci
on: Sea AO2 B = 2, entonces ACB = y por hipoteis AO1 B = 2 + 70. Por
angulo seminscrito, ABE = + 35, y como el 4ABE es isosceles, AEB = + 35.
Finalmente, por la formula del angulo externo aplicada al 4BCE, EBC = AEB
ECB = 35
12. Dos circunferencias 1 y 2 se cortan en A y B. Una recta por A corta a 1 y 2 en C y
D, respectivamente, y la paralela a CD por B corta 1 y 2 en E y F , respectivamente.
(Ver figura 125). Demuestre que 4CDB 4EAF .
Soluci
on: Sean G = AE BC y H = AF BD. Como AC k BE y ACEB es ccli38
co, CAG = GEB = ACG = GBE = ; analogamente, DAH = HF B =
ADH = F BH = . Observe que 4GAC y 4GBE son triangulos isosceles y por tanto AE = AG+GE = CG+GB = CB; de forma similar se obtiene AF = DB. Finalmente,
EAF = 180 = CBD, por lo que, por el criterio LAL, 4CBD 4EAF .
13. La Recta de Simson-Wallace. Sean X, Y y Z los pies de las alturas trazadas desde un
punto P en el circuncrculo del 4ABC hacia AB, BC y CA, respectivamente. Demuestre
que X, Y y Z estan alineados.39
Soluci
on: Como BP Y X es cclico, Y XP = Y BP = . Como ABP C es cclico,
CBP = CAP = . Como AXP Z es cclico, ZAP = ZXP = . Por lo tanto,
dado que Y XP = ZXP , los puntos X, Y y Z estan alineados.
38

ACEB es un trapecio is
osceles.
El recproco tambien es cierto, si X, Y y Z estan alineados, entonces P debe estar sobre el circuncrculo
del 4ABC; en cualquier otro caso, el 4XY Z se llama el tri
angulo pedal con respecto al punto P .
39

89

Figura 125

Figura 126: Recta de Simson-Wallace


14. Sea P un punto exterior al cuadrado ABCD tal que AP C = 90 , Q es la interseccion
de AB y P C, y R el pie de la perpendicular por Q a CA. Demuestre que P , R y D estan
alineados.
Soluci
on: Como AP C + ADC = 180, el cuadrilatero P ADC es cclico, entonces
AP D = ACD = 45. Analogamente, como AP Q + ARQ = 180, el cuadrilatero
P ARQ es cclico, entonces AP R = AQR = 90QAR = 45. Por lo tanto, como
AP D = AP R, los puntos P , R, D estan alineados. Figura 127.
15. (OIM 2002, P-4) En un triangulo escaleno ABC se traza la bisectriz interior BD, con D
sobre AC. Sean E y F puntos sobre la recta BD tales que (AE k CF ) BD, y sea M el
punto sobre el lado BC tal que DM BC. Demuestre que EM D = DM F . Figura
128.
90

Figura 127
Soluci
on: Como DM M C y DF F C, DF CM es cclico, por lo tanto DM F =
DCF = , y como AE k F C, entonces EAD = DCF = . Sea G la interseccion
de AE con BC. Como AG BE, BE es altura y bisectriz del 4ABG, por lo que este
triangulo es isosceles y ademas BE es mediatriz de AG; entonces EGD = EAD = .
Y finalmente, podemos ver que DEM G es cclico, pues DEG = DM G = 90 , as que
EM D = EGD = . De aqu, el resultado es inmediato.

Figura 128
16. (OMCC 2003, P-2) Sea S una circunferencia y AB un diametro de ella. Sea t la recta
tangente a S en B y considere dos puntos C y D en t tales que B este entre C y D. Sean
E y F las intersecciones de S con AC y AD y sean G y H las intersecciones de S con
CF y DE. Demuestre que AH = AG.
Soluci
on: Como AEBF es cclico (Ver figura 129), AEF = ABF . Luego, como AB
CD y BF AD, se cumple tambien ABF = F DB, por lo que AEF = F DC, es
decir, el cuadrilatero CDF E es cclico. Utilizando este resultado y el hecho que EGHF
tambien es cclico, se tiene EDC = EF G = EHG, por lo que CD k GH. Esto
91

implica que AB GH, y como AB pasa por el centro de la circunferencia, debe ser
mediatriz de GH, por lo tanto AG = AH.

Figura 129
17. (The 59th Romanian Mathematical Olympiad District Round) Considere un cuadrado
ABCD y un punto E sobre el lado AB. La diagonal AC corta al segmento DE en el punto P . La perpendicular por P a DE corta al lado BC en F . Probar que EF = AE + CF .
Soluci
on: Se construye E 0 sobre BC de tal manera que CE 0 = AE (como se muestra en la figura 130) y que C quede entre F y E 0 , as por LAL se tiene que los triangulos
rectangulos 4DAE 4DCE 0 por lo tanto ADE = CDE 0 luego EDE 0 = 90. Por
otra parte, el cuadrilatero DCF P es cclico, por lo que P DF = P CF = 45 entonces
F DE 0 = EDE 0 EDF = 45. Ahora por LAL los triangulos 4DEF 4DE 0 F ,
por lo que EF = E 0 F = E 0 C + CF = AE + CF .

Figura 130

92

18. Teorema de Arqumedes: En la figura 131, la region delimitada por tres semicircunferencias mutuamente tangentes, es conocida como cuchilla de zapatero o arbelos. Demostrar
que las circunferencias sombreadas son congruentes.

Figura 131: Teorema de Arqumedes.


Soluci
on: Sean AB, AC, BC los diametros de las semicircunferencias que forman el arbelos, de radios r, r1 , r2 y centros O, O1 , O2 , respectivamente. De momento nos concentramos
en el lado izquiero de la figura Referenciasfigura58; sea C1 el centro de la circunferencia de
la izquierda y R1 su radio; D, E y F son los puntos de tangencia de esta circunferencia con
dos semicircunferencias del arbelos y con la recta perpendicular a los diametros por C; finalmente, G es la proyeccion de C1 sobre AB. En primer lugar, OO1 = OAO1 A = rr1 .
Por otra parte, observe que O, C1 , D estan alineados y O1 , C1 , E tambien estan alineados,
entonces OC1 = OD C1 D = r R1 y O1 C1 = O1 E + EC1 = r1 + R1 . Ademas, como
CF C1 G es un rectangulo, GC = F C1 = R1 , entonces O1 G = O1 C GC = r1 R1 y
OG = O1 G O1 O = (r1 R1 ) (r r1 ) = 2r1 (r + R1 ). Ahora, aplicando el teorema
de Pitagoras a 4GO1 C1 y 4GOC1 se tiene
O1 C12 O1 G2 C1 G2 = OC12 OG2 C1 G2
(r1 + R1 )2 (r1 R1 )2 = (r R1 )2 (2r1 (r + R1 ))2
4r1 R1 = 4rR1 4r12 + 4r1 r + 4r1 R1
r1 (r r1 )
R1 =
r

Figura 132
Analogamente, si r2 y R2 son los radios del semicrculo y del crculo de la derecha, respectivamente, entonces
r2 (r r2 )
R2 =
r
93

Pero 2r = AB = AC + BC = 2r1 + 2r2 , entonces r2 = r r1 , y sustituyendo en la


ecuacion anterior se tiene
R2 =

(r r1 ) (r (r r1 ))
(r r1 )r1
=
= R1
r
r

94

Teorema de Thales y Semejanza.


1. (IV OMCC, P-4) Sea ABC un triangulo, D el punto medio de BC, E un punto sobre
el segmento AC tal que BE = 2AD y F el punto de interseccion de AD con BE. Si
CAD = 60, encuentre la medida de los angulos del 4F EA. Figura 133.
Soluci
on: Se traza por D una paralela a BE y sea G el punto por el que esta paralela
= AD;
corta al lado AC. Como DG es base media del 4BCE se tiene que DG = BE
2
entonces 4ADG es isosceles y tiene un agulo de 60, por lo que debe ser equilatero.
Finalmente 4AEF tambien es equilatero, por tanto sus angulos son iguales a 60.

Figura 133
2. Sea ABCD es un trapecio con AD k BC. M y N son los puntos medios de CD y BC,
M
= 14 , demuestre que
respectivamente, y P el punto com
un de las rectas AM y DN . Si PAP
ABCD es paralelogramo.
Soluci
on: Sea Q el punto medio de DN , entonces QM k BC k DA. Como M Q es
= CB
. Por otra parte, como 4P M Q ' 4P AD,
base media del 4CDN , M Q = CN
2
4
MQ
PM
1
AD
= AP = 4 , entonces M Q = 4 . Finalmente, como BC k DA y BC = DA, ABCD
AD
es paralelogramo.
3. En la figura 134, BC = CD = DE = EA = x y AEB = 90. Demuestre que ABC +
ACD + ADE = 90.

Figura 134

Soluci
on 1: Por Pitagoras, AD = 2x. Observe que DA2 = 2x2 = DB DC, por lo que
4ABD ' 4CAD; entonces ABD = CAD y por tanto ABC + ACD + ADE =
95

CAD + ACD + ADE = 2ADE = 90.


Soluci
on 2: Considere la siguiente cuadrcula (Figura 135). Observe que al hacer una
rotacion de centro A y angulo igual a 90, el segmento AC se transforma en AF , por lo
que el 4ACF es triangulo rectangulo isosceles, y ADE = ACF . Se cumple ABC =
CF D, porque se forman con la diagonal de tres cuadrados; analogamente, ACD =
AF D, porque se forman con la diagonal de dos cuadrados. Sumando los angulos internos
del 4ACF se obtiene el resultado buscado.

Figura 135
4. (Asiatico Pacfica) Sea ABC un triangulo y D el pie de la altura con respecto a A. Sean E
y F puntos en una recta que pasa por D (distintos de D) tales que AE CE y AF BF .
Sean M y N los puntos medios de BC y EF , respectivamente. Demuestre que AN N M .
Soluci
on: En la figura 136, AE CE y AD DC entonces, ADEC es cclico, as que
DEA = DCA. Del mismo modo, como AF BF y AD DB, AF BD es cclico
y entonces AF D = ABD. Esto implica que 4ABC ' 4AF E y a partir de esta
semejanza, 4ABM ' 4AF N . Luego, AM B = AN F , por lo que el cuadrilatero
AN DM es cclico, y por lo tanto AN M = ADM = 90.

96

Figura 136

Puntos y rectas notables.


1. De acuerdo con los datos de la grafica 137, calcular el valor de AB.

Figura 137
AC
CD
Soluci
on 1: Por el teorema de la bisectrz AB
= DB
, de donde AC = 45 x, luego, aplicando
2
el teorema de pitagoras al 4ABC, se tiene que x2 + 182 = 54 x , que despues de resolver
se tiene que x = 24.

Soluci
on 2: Dado que D es un punto de la bisectrz del BAC, entonces D equidista
de los lados de dicho angulo, sea pues H AC talque DHAC y DH = DB = 8
entonces, aplicando el teorema de pitagoras en el 4CDH se deduce que HC = 6, por lo
que AC = x + 6, y por el teorema de pitagoras en el 4ABC, x2 + 182 = (x + 6)2 , por lo
tanto, x = 24.
2. Sea ABCD un paralelogramo. Q es el punto medio de AD, F el pie de la perpendicular
por B sobre QC. Probar que AF = AB.
Soluci
on: Sea E el punto medio de BC y G la interseccion de AE con BF . Como
AE k CQ, se tiene que AG BF . Pero tambien, como AE k CQ, entonces EG k CF
por lo que en el 4BCF , EG es base media. Entonces BG = GF de donde se sigue que
4ABF es isosceles porque BG es altura y mediana.
97

3. Sea ABCD un cuadrilatero tal que AB = CD. Las mediatrices de AC y BD se cortan


en P . Probar que P AC = P CA = P BD = P DB. Figura 138.
Soluci
on: Como P esta sobre las mediatrices de AC y BD, P A = P C y P B = P D,
y por hipotesis, AB = CD, entonces por criterio LLL, 4ABP 4CDP . De aqu,
AP B = CP D, entonces BP D = AP D + AP B = AP D + CDP = AP C;
por lo tanto, 4BP D ' 4CP A, dada la igualdad anterior y el hecho que son triangulos
isosceles. De esta semejanza se obtiene P AC = P CA = P BD = P DB.

Figura 138
4. ABC es un triangulo y P un punto en su interior. Sean A0 , B 0 y C 0 las reflexiones de
P sobre BC, CA y AB, respectivamente. D, E y F son los pies de las perpendiculares
respectivos desde A, B y C hacia B 0 C 0 , C 0 A0 y A0 B 0 . Probar que AD, BE y CF son
concurrentes. Figura 139.
Soluci
on: Por propiedades de reflexion axial AC 0 = AP = AB 0 , por lo que el 4AB 0 C 0 es
isosceles, y entonces AD es mediatriz de B 0 C 0 . Analogamente, BE es mediatriz de C 0 A0 ,
mientras que CF es mediatriz de A0 B 0 . Por lo tanto, las rectas AD, BE, CF concurren
en el circuncentro del 4A0 B 0 C 0 .
5. (Arnoldo Aguilar) En la figura 140, ABGH, BCF G y CDEF son cuadrados. Si I es el
centro de ABGH y J = DH BG, demuestre que I, J y F estan alineados.
Soluci
on: Como G es punto medio de HF , BG es una mediana del 4BF H. Ademas,
BDF H es un paralelogramo, luego sus diagonales BF y DH se cortan en su punto medio,
digamos K. Se sigue que HK es tambien una mediana del 4BF H, y en consecuencia el
punto de corte de J = KH BG es el centroide del 4BF H. Pero I es el punto medio de
BH, as que F I es la tercera mediana del 4BF H, por lo tanto J esta sobre el segmento
F I.

98

Figura 139

Figura 140
6. (Arnoldo Aguilar) Sea ABC un triangulo equilatero. M y N son los puntos medios de AB
y BC, respectivamente. Exteriormente al 4ABC se construye un triangulo rectangulo
isosceles 4AP C, con AP C = 90 . Si I es la interseccion de AN y M P , demuestre que
CI es la bisectriz de ACM .

Soluci
on: Observe que AN es bisectriz del BAC.
Como AP C = BM C = 90, el cuadrilatero
AP CM es cclico, por lo que P M C = P AC =
P CA = P M A = 45, entonces M P es bisectriz
del AM C. De aqu se concluye que I = M P AN
es el incentro del 4ACM , por lo que CI es bisectriz
del ACM .

7. En la figura 141, el 4ABC es tal que A = 90 y B = 60. Cual es el radio de la


circunferencia?

Soluci
on: Por relaciones de triangulos notables, BC = 2 y CA = 3. Sean P y Q
las proyecciones de O sobre AB y AC respectivamente; por construccion, AP OQ es un
rectangulo, pero como OP = r = OQ, es tambien cuadrado, por lo que AP = r. Observe
99

Figura 141
que la circunferencia es el excrculo del 4ABC, por lo que AP = s y entonces
r = s

1+2+ 3
=
2
3+ 3
=
2
8. Dado el paralelogramo ABCD, sea M el punto medio de AB, y N la interseccion de CD
con la bisectriz interna del ABC. Demuestre que M C BN si y solo si AN es bisectriz
del DAB.
Soluci
on:
() Si suponemos que M C BN entonces BN es mediatriz de M C, y como BM k CN
entonces CBN = M BN = CN B = M N B, esto implica que BC k M N , y por
tanto N es punto medio de CD; as, AM N D es un rombo y AN es bisectriz del DAM .
() Si suponemos que AN es bisectriz del DAB, es propiedad conocida que AN BN ,
por lo que M es el circuncentro del 4ABN y por la relacion entre angulo central y angulo
inscrito se tiene AM N = 2ABN = ABC, por lo tanto M N k BC y BCN M es un
rombo, de donde se obtiene M C BN .
9. En el 4ABC, se sabe que los vertices B, C, el circuncentro O y el ortocentro H del
4ABC estan todos sobre una misma circunferencia. Figura 142.
a) Calcule el valor de A.
b) Demuestre que el incentro tambien pertenece al circuncrculo de BCOH.
Soluci
on:
a) Sea BAC = . Como O es el circuncentro del 4ABC, tenemos que BOC = 2. Por
otra parte, sabemos que al ser H ortocentro, se cumple que BHC = 180 . Ahora
100

bien, la condicion de que B, C, H y O son concclicos implica que BOC = BHC,


de donde 2 = 180 , y por tanto = 60 .

Figura 142
b) Este problema se basa en el siguiente resultado: si I es el incentro del 4ABC entonces
BIC = 90 + A
. Como en este caso A = 60, entonces BIC = 120 = BOC =
2
BHC, por lo que B, C, O, H, I, se ubican sobre una misma circunferencia.
10. Sea ABC un triangulo tal que las medianas respectivas a B y C son perpendiculares.
Demuestre que se cumple la relacion (Ver figura 143).
5BC 2 = CA2 + AB 2 .

Figura 143
Soluci
on: Sean BB 0 y CC 0 las medianas que son perpendiculares, y sea G el centroide.
Observe que el cuadrilatero BCB 0 C 0 tiene diagonales perpendiculares; por el teorema de
Pitagoras se cumple
BC 2 + B 0 C 02 = C 0 B 2 + B 0 C 2

2

2 
2
BC
AB
AC
2
=
+
BC +
2
2
2
2
2
2
5BC = AB + AC
101

11. Sea ABC un triangulo de ortocentro H. Sean P y Q los pies de las perpendiculares desde
H a las bisectrices interior y exterior de A, respectivamente. Si M es el punto medio de
BC, mostrar que P , Q y M estan alineados. Figura 144.
Soluci
on: Sean E y F los pies de las alturas trazadas desde a B y C, respectivamente.
Se sabe que AP AQ, por lo que AP HQ es un rectangulo. Como AP H = AQH =
AEH = AF H = 90, los puntos P , Q, E, F , pertenecen a una circunferencia de
diametro AH. Ademas, en esta circunferencia, como AP y AQ son bisectrices (interior
y exterior, respectivamente) del EAF , P y Q son los puntos medios del arcos EF ,
por lo que P Q es la mediatriz de EF . Por otra parte, como BEC = BF C = 90, el
cuadrilatero BCEF es cclico, y el circuncentro es M , por lo que M E = M F ; entones M
esta en la mediatriz de EF , la cual es P Q.

Figura 144
12. En un triangulo ABC, sea M el punto medio de BC. Si se cumple que AB 6= AC y
ademas M AC + ABC = 90 , hallar BAC. Figura 145.
Soluci
on: Sin perdida de generalidad, suponga que AB > AC. Sea N la interseccion de
AB con la mediatriz de BC. Se forma el 4BCN que es isosceles, entonces CN M =
90 M CN = 90 M BN = CAN , lo cual implica que el cuadrilatero ACM N es
cclico. Por lo tanto, BAC = BM N = 90.
13. Sea ABC un triangulo y U un punto de su circuncrculo tal que AU es bisectriz. Las
mediatrices en AB y AC cortan a AU en X y Y . Sea T la interseccion de BX con CY .
Demostrar que AU = T B + T C. Figura 146.
Soluci
on:40 Como X y Y pertenecen a las mediatrices de AB y AC, respectivamente,
y a la bisectriz AU , entonces 4ABX y 4ACY cumplen ser isosceles y semejantes entre
si, porque XBA = XAB = Y AC = Y CA = . Esto implica T XY = XBA +
40

El caso cuando AB = AC es trivial, porque X, Y y T colapsan en el circuncentro del 4ABC.

102

Figura 145
XAB = 2 = Y AC+Y CA = T Y X, es decir, el 4T XY es isosceles con T X = T Y .
Por otra parte, como ABU C es cclico, U BC = U AC = U AB = U CB = . De
aqu se concluye que 4U BC es isosceles, con U B = U C. Ademas, XU B = ACB =
Y CU y XBU = ABC = Y U C; por criterio ALA, 4U XB 4CY U , por lo que
BX = Y U . Finalmente, T B + T C = (BX T X) + (CY + T Y ) = Y U + AY = AU .

Figura 146
14. (The 59th Romanian Mathematical Olympiad Final Round) Sea ABCD un rectangulo de
centro O con AB 6= BC. La perpendicular en O a BD corta a las lneas AB y BC en los
puntos E y F , respectivamente. Sean M y N los puntos medios de los segmentos CD y
DA, respectivamente. Probar que las lneas rectas F M EN .
Soluci
on: Considere la figura 147, sin perdida de generalidad, se ha supuesto AB < BC.41
Sea L el punto medio de AB, y H es la interseccion de EF con AD. Se tiene que
LN k BD, y como BD EF entonces LN EF ; ademas, como ABCD es un rectangulo, DA AB, por lo tanto, H es el ortocentro del 4ELN , y as, LH EN . Por otra
parte, las reflexiones de L y H con respecto a O son respectivamente M y F , por lo que
LH k M F , lo cual implica que F M EN .
41

El otro caso es completamente an


alogo.

103

Figura 147
15. Sea ABC un triangulo rectangulo, con A = 90 . Sea D un punto en su interior tal que
DAC = DCA = DBC = , y AC = BD. Determine el valor de . Figura 148.
Soluci
on: Sean P y Q los pies de las perpendiculares trazadas desde D hacia CA y AB,
respectivamente, R es un punto sobre BC tal que DB DR, y E es la interseccion de CD
con AB. Como el 4ACD es isosceles, P es punto medio de AC, entonces AC = 2P A =
2DQ = BD, por lo que el 4BDQ es un triangulo notable y DBQ = 30. Por otra parte,
por criterio ALA, 4ACE 4DBR, por lo que CE = BR; como P D k AE, D es punto
medio de CE; as, si M es el punto medio de BR (y circuncentro del 4BDR) se cumple
que DC = RM = DM , por lo que el 4CDM es isosceles. Por la relacion entre el angulo
inscrito y el angulo central DM R = 2DBR, por lo tanto DCR = 2. Sumando los
angulos internos del 4ABC se tiene A + B + C = 90 + 30 + + 3 = 180, lo cual
implica = 15.

Figura 148
16. Sea ABC un triangulo y M un punto tal que M AB = 10, M BA = 20, M AC = 40
104

y M CA = 30. Probar que el 4ABC es isosceles. Figura 149.


Soluci
on: Sea D la reflexion del punto A con respecto a la recta BM . Entonces el 4AM D
es isosceles con AM D = 2 (M AB + ABM ) = 60 y por lo tanto es equilatero. Tambien DBA = 2M BA = 40 y como BAC = 50, implica que DB AC. Sea E la
interseccion de BD con CM , se cumple que CED = 90 ACE = 60 = M AD,
por lo que el cuadrilatero AM ED es cclico. De aqu, DEA = DM A = 60. Como
DEC = DEA y ED AC, se tiene que ED es bisectriz y altura en el 4AEC, por
lo tanto ED es mediatriz de AC, lo cual implica que BA = BC.

Figura 149
17. Teorema de Poncelet: Demuestre si 4ABC es un triangulo rectangulo con A = 90
entonces 2(r + R) = b + c.

Figura 150: Teorema de Poncelet


Soluci
on: Sean O e I el circuncentro y el incentro del 4ABC. Como A = 90, O es el
punto medio de BC, por lo que a = 2R. Por otra parte, si P y Q son las proyecciones de I
sobre AB y AC, claramente AP IQ es rectangulo, pero como I es incentro IP = r = IQ,
por lo que AP IQ es cuadrado. Se sabe que para un triangulo cualquiera AP = s a, por
lo tanto
r = sa
b+ca
r =
2
2r + a = b + c
2(r + R) = b + c
105

18. En la figura 151, ABCD y P QRS son cuadrados, 4ABP 4BCQ 4CDR 4DAS
y los los radios de las cinco circunferencias son iguales a r. Si a es el lado del cuadrado
ABCD, determine r en funcion de a.

Figura 151
Soluci
on: Se tiene AB = a y se definen b = AP y c = BP ; observe que por las congruencias BQ = b, por lo que P Q = cb = 2r. Por otra parte (analogamente a la demostracion
del teorema de Poncelet), al calcular el inradio del 4ABP se tiene que 2r = b + c a,
entonces c b = b + c a, lo cual implica que a = 2b. Por lo tanto, el 4ABP es un
triangulo notable de 30, 60, 90, y as
cb
2

3
a 12 a
= 2
2
!

31
=
a
4

r =

106

19. Recta de Euler. El centroide G, el ortocentro H y el circuncentro O de un triangulo


estan alineados, y ademas GH = 2GO.

Figura 152: Recta de Euler


Soluci
on: Considere la siguiente figura. Sean AHa y BHb alturas, OA0 y OB 0 mediatrices.
Observe que HA BC y OA0 BC, por lo que HA k OA0 ; analogamente HB k OB 0 ;
tambien, por el teorema de la base media AB k A0 B 0 y AB = 2A0 B 0 . Esto implica que
4ABH ' A0 B 0 O y la razon de semejanza es 2; en particular AH = 2A0 O. Si definimos
G como la interseccion de la mediana AA0 con HO, claramente 4AHG ' 4A0 OG y la
razon de semejanza es la misma que la anterior, por lo que GA = 2GA0 , i.e., G es el
centroide del 4ABC. Esto implica que el ortocentro, el centroide y el circuncentro de un
triangulo estan alineados, y por la semejanza GH = 2GO.

107

Vous aimerez peut-être aussi